• Shuffle
    Toggle On
    Toggle Off
  • Alphabetize
    Toggle On
    Toggle Off
  • Front First
    Toggle On
    Toggle Off
  • Both Sides
    Toggle On
    Toggle Off
  • Read
    Toggle On
    Toggle Off
Reading...
Front

Card Range To Study

through

image

Play button

image

Play button

image

Progress

1/101

Click to flip

Use LEFT and RIGHT arrow keys to navigate between flashcards;

Use UP and DOWN arrow keys to flip the card;

H to show hint;

A reads text to speech;

101 Cards in this Set

  • Front
  • Back
A 20-year-old male calls you on the day of his appointment saying that he must cancel. He reports that he has suddenly developed a high fever, a severe headache, and has been vomiting. He sounds confused over the phone. Noting these symptoms, you begin to suspect your patient may have meningitis and should seek medical help. Which of the following cells increases the difficulty of treatment for meningitis due to the role it plays in creating the blood brain barrier?
A. Ependymal cell
B. Oligodendroglia
C. Astroglia
D. Schwann Cells
E. The blood brain barrier does not make treating meningitis any more complicated than any other bacterial infection.
Answer 1
A. Ependymal cell – While these cells do line and protect the brain, they are not a part of the blood brain barrier. – WHAT BARRIER ARE THEY A PART OF???
B. Oligodendroglia – These cells produce myelin in the CNS and do not contribute to the blood brain barrier.
C. Astroglia –CORRECT. Astrocytes make up PART OF the blood brain barrier, which prevents certain compounds, including many antibacterial medications, from accessing the brain. This means that doctors often have to carefully choose the medications that they use to treat meningitis.
D. Schwann Cells – these cells only exist in the PNS and they produce myelin; as a result, they are not associated with the blood brain barrier and should not be a barrier to the brain and CNS.
E. The blood brain barrier does not make treating meningitis any more complicated than any other bacterial infection. – The blood brain barrier prevents some drugs from entering the brain, which is where the meningitis exists (ACTUALLY THAT WOULD BE ENCEPHALITIS). This makes selecting the proper treatment for meningitis much more difficult, as the selected drug must be able to pass through the blood brain barrier.

Once you include confusion into the signs for infection, the diagnosis becomes encephalitis which is a more appropriate diagnosis for your case. I will adjust the wording. Good question overall.
A 32-year-old female is visiting you for treatment of her sprained ankle. She mentions that she has recently become pregnant and is concerned that her baby will develop spina bifida, as there is a history of the condition in her family. She is wondering if you, as a medical professional, happen to know anything about spina bifida, and what she could do to reduce the chances of her child developing it. What might you tell her?
A. Correct nutritional levels of folate may help reduce the risk.
B. Reduce her alcohol intake to reduce the risk.
C. Eat fatty foods to help boost myelin production in the baby.
D. Eat lots of protein to ensure healthy development of somites in the embryo.
E. Spina bifida is only developed via genetic means, so there isn’t much she can do to prevent this.
Answer 2
A. Correct nutritional levels of folate may help reduce the risk. – CORRECT. Folate has been shown to reduce the risk of an embryo developing spina bifida during gestation, although the mechanism through which this occurs is not well understood. Mothers should increase their folate intake prior to and during pregnancy for the decreased risk.
B. Reduce her alcohol intake to reduce the risk. - This is more related to the prevention of fetal alcohol syndrome than spina bifida, although admittedly it still might be worth mentioning. (THIS CAN BE CONFUSING AND ALCOHOL MIGHT BE RELATED TO NTD’S AS WELL.) I’LL REWRITE THIS ANSWER CHOICE.
C. Eat fatty foods to help boost myelin production in the baby. – Spina bifida is not related to inadequate myelination; rather, it is the result of the neural tube not closing during development.
D. Eat lots of protein to ensure healthy development of somites in the embryo. – Spina bifida is linked to incomplete formation of the neural tube, which developed from the ectoderm. Somites develop from the mesoderm, thus this answer is incorrect.
E. Spina bifida is only developed via genetic means, so there isn’t much she can do to prevent this. – Spina bifida is linked to both genetic and environmental factors, including inadequate folate in the mother’s diet during development.
A 24-year-old patient is referred to your neurology clinic with a prescription to strengthen her wrist after six weeks of immobilization in a cast. While taking her history she tells you that she fractured her wrist after she fell in her living room. She reports that she has had many balance issues, so you do a Rhomberg test, suspecting possible cerebellar impairment. An impairment of the cerebellum involves the adult derivative of the:
A. Telencephalon
B. Diencephalon
C. Metencephalon
D. Mesencephalon
E. Myelencephalon

Rhomberg is not a specific test for the cerebellum. I will modify the wording. Good question, otherwise.
Answer 3
A. Telencephalon- The telencephalon becomes the superior portion of the forebrain.
B. Diencephalon- The diencephalon is superior to the cerebellum in the forebrain.
C. Metencephalon- CORRECT. The cerebellum is derived from the metencephalon of hindbrain.
D. Mesencephalon- The mesencephalon becomes the midbrain, which is superior to the cerebellum.
E. Myelencephalon- The myelencephalon becomes the medulla oblongata.
You are seeing a patient in a hospital 2 days after a motor vehicle accident. The patient had a head injury and now reports that he can no longer smell anything. You suspect an injury to Cranial Nerve I or some type of injury to the Paleocortex. The damaged area is the adult derivative of:
A. Telencephalon
B. Diencephalon
C. Mesencephalon
D. Midbrain
E. Myelencephalon
Answer 4
A. Telencephalon- CORRECT. Cranial Nerve I and the paleocortex (Olfactory cortex) are derived from the telencephalon of the forebrain.
B. Diencephalon- Associated with the Optic Nerve
C. Mesencephalon- Associated with Oculomotor Nerve and Trochlear Nerve
D. Midbrain- Associated with Oculomotor Nerve and Trochlear Nerve
E. Myelencephalon- Associated with Trigeminal, Abducens, Facial, Vestibulocochlear, Glossopharyngeal, Vagus, and Hypoglossal nerves

Two of the same choice??? I know you want to be nice and all…
You have an 8-year-old patient who has come to outpatient physical therapy to work on gait training. The patient has a past medical history of spina bifida and you observe a tuft of hair on her sacral region. An abnormality in the development of which of the following structures is associated with this disease?
A. Notocord
B. Posterior neuropore
C. Somites
D. Endoderm
E. Anterior neuropore
Answer 5
A. Notocord- The notocord is non-neural tissue that is the precursor for development of the nervous system. Its remnants may exist in the nucleus pulposus, but it does not become the spinal cord.
B. Posterior neuropore- CORRECT. Incomplete closure of the posterior neuropore can result in spina bifida.
C. Somites- somites become the musculoskeletal system, not the spinal cord.
D. Endoderm- becomes GI, respiratory, and urinary systems. It does not become the spinal cord.
E. Anterior neuropore- Anencephaly is a disease that occurs when the anterior neuropore does not completely close.

Excellent question.
You are performing an initial evaluation on a 25-year-old male complaining of right hand weakness. You decide to perform an upper quarter screen and find weak elbow flexors and wrist extensors, and sensory loss to the thumb. You suspect a problem at the C6 nerve root level based on the dermatomal sensation loss and myotomal weakness. Dermatomes and myotomes are derived from:
A. Notocord
B. Ectoderm
C. Endoderm
D. Somites
E. Magic
Answer 6
A. Notocord - precursor to the development of the nervous system and remnants only remain in the nucleus pulposus.
B. Ectoderm - becomes the skin and nervous system.
C. Endoderm - becomes the lining of hollow organs.
D. Somites – CORRECT. Somites are derived from the mesoderm and become the sclerotomes, myotomes, and dematomes.
E. Magic - Do you believe in magic?

Great question… and finally a joke that made me laugh!
You are treating a 60-year-old patient on the first day of your acute care clinical affiliation. As you are introducing yourself to her, you notice the left side of her face seems to be drooping. When asked, she tells you that she has been diagnosed with Bell’s Palsy. What is the derivative of the nerve responsible for Bell’s Palsy?
A. Telencephalon
B. Mesencephalon
C. Diencephalon
D. Omniencephalon
E. Metencephalon
Answer 7
A. Telencephalon - Olfactory nerve - not associated with Bell’s palsy.
B. Mesencephalon - Oculomotor and Trochlear aren’t responsible for this condition.
C. Diencephalon - Optic nerve lesions don’t cause facial hemiparalysis.
D. Omniencephalon - only present in embryonic aliens.
E. Metencephalon – CORRECT. The facial nerve (CN VII) derives from this embryonic component.

Excellent question and again funny!
While on your acute care affiliation, you find yourself treating a patient in the cardiac care unit. Everything seems to be going smoothly until, all of a sudden, your patient codes. At this time, which of the following effects induced by the sympathetic nervous system would you experience?
A. Increased Heart Rate
B. Increased Blood Pressure
C. Increased Intestinal & Gland Activity
D. A & B
E. All of the above
Answer 8
A. Increased Heart Rate: this is a response of the sympathetic NS but it is not the only correct answer choice.
B. Increased Blood Pressure: this is a response of the sympathetic NS but it is not the only correct answer choice.
C. Increased Intestinal & Gland Activity: this is a response of the parasympathetic nervous system.
D. A & B (Increased Heart Rate & Increased BP) – CORRECT. The sympathetic nervous system induces both an increase in HR and an increase in blood pressure.
E. All of the above: not the case because answer C is a response of the parasympathetic nervous system.

Good question. I will modify the wording because I don’t use “all of the above” or A & B type questions.
While on your acute care affiliation, you find yourself treating a patient in the cardiac care unit. Everything seems to be going smoothly until, all of a sudden, your patient codes. As you wait for the emergency response team to arrive to treat your patient who has just coded, you begin performing CPR. First, you “Look, listen, and feel” and begin compressions. You feel for a pulse & your patient is confirmed pulseless. You then begin compressions and adjust your pressure based upon sensory feedback to attain appropriate compression depth. This sensory information travels in the spinal cord via:
A. Efferent fibers in ventral rootlets
B. Efferent fibers in dorsal rootlets
C. Afferent fibers in ventral rootlets
D. Afferent fibers in dorsal rootlets
E. This is a life or death situation! CPR is no joke—it uses both afferent & efferent fibers to send sensory information to the spinal cord.
Answer 9
A. Efferent fibers in ventral rootlets: incorrect—Efferent fibers are motor fibers that leave the spinal cord via ventral rootlets.
B. Efferent fibers in dorsal rootlets: incorrect—Afferent signals travel in dorsal rootlets.
C. Afferent fibers in ventral rootlets: incorrect—Efferent signals travel in ventral rootlets.
D. Afferent fibers in dorsal rootlets: CORRECT. —Sensory information sends an afferent signal that travels into the dorsal rootlets of the spinal cord.
E. This is a life or death situation! CPR is no joke—it uses both afferent & efferent fibers to send sensory information to the spinal cord. —Incorrect. Though CPR is important, efferent fibers are motor fibers and travel via ventral rootlets, while afferent fibers are sensory fibers that travel via dorsal rootlets.

Great question
Question 10
While on your acute care affiliation, you find yourself treating a patient in the cardiac care unit. Everything seems to be going smoothly until, all of a sudden, your patient codes. As you wait for the emergency response team to arrive to treat your patient who has just coded, you begin performing CPR. First, you “Look, listen, and feel.” You determine that you need to perform compressions, a motor response. This motor information travels out of:
A. Efferent fibers in ventral rootlets
B. Efferent fibers in dorsal rootlets
C. Afferent fibers in ventral rootlets
D. Afferent fibers in dorsal rootlets
E. This is a life or death situation! CPR is no joke—it uses both afferent & efferent fibers to send sensory information to the spinal cord.
Answer 10
A. Efferent fibers in ventral rootlets: CORRECT. —Efferent fibers are motor fibers that leave the spinal cord via ventral rootlets.
B. Efferent fibers in dorsal rootlets: incorrect—Afferent signals travel in dorsal rootlets.
C. Afferent fibers in ventral rootlets: incorrect—Efferent signals travel in ventral rootlets.
D. Afferent fibers in dorsal rootlets: incorrect—Sensory information sends an afferent signal that travels into the dorsal rootlets of the spinal cord.
E. This is a life or death situation! CPR is no joke—it uses both afferent & efferent fibers to send sensory information to the spinal cord. —Incorrect. Though CPR is important, efferent fibers are motor fibers and travel via ventral rootlets, while afferent fibers are sensory fibers that travel via dorsal rootlets

Good question… now its getting repetitive  I’ll use one for the midterm and one for the final.
Lecture 7: The Autonomic Nervous System

Question 1
You are currently completing a six-minute walk test with a 65-year-old male patient to assess his exercise capacity. During the test, his heart rate increases. Later, while writing his treatment notes, you reflect on the neuroanatomy of this effect (increased heart rate during exercise) of the autonomic nervous system. What neurotransmitter would be released by the postganglionic cell in this situation and where would the ganglion associated with this effect be located?
A. Neurotransmitter: Acetylcholine; Location: very close to the heart.
B. Neurotransmitter: Acetylcholine; Location: close to the spinal cord.
C. Neurotransmitter: Norepinephrine; Location: very close to the heart.
D. Neurotransmitter: Norepinephrine; Location: close to the spinal cord.
E. Neurotransmitter: Substance P; Location: in the Myenteric Plexus.
Answer 1
A. Neurotransmitter: Acetylcholine; Location: very close to the heart. – This combination might be true if a parasympathetic response were occurring; however, this would involve a slowing of the heart rate, so this is incorrect.
B. Neurotransmitter: Acetylcholine; Location: close to the spinal cord. – The ganglion location is correct for a sympathetic response, however the neurotransmitter is incorrect. A preganglionic cell would release acetylcholine, however.
C. Neurotransmitter: Norepinephrine; Location: very close to the heart. The neurotransmitter is correct for a sympathetic response; however, the ganglion location is more representative of a parasympathetic ganglion.
D. Neurotransmitter: Norepinephrine; Location: close to the spinal cord. – CORRECT. The patient is exhibiting a sympathetic response, which leads to a release of norepinephrine rather than acetylcholine by the postganglionic cell. Furthermore, sympathetic ganglia are located closer to the spinal cord than their target organs - parasympathetic ganglia are located very near to their target organs.
E. Neurotransmitter: Substance P; Location: in the Myenteric Plexus. – The enteric division of the autonomic nervous system, which does not act on the heart, utilizes these attributes.

Great question
You are about to evaluate and treat a 25-year-old female who reports to therapy with an ACL tear. You note that her past medical history includes pheochromocytoma, which has been diagnosed but not yet treated. What effects might you observe that could be explained by this condition?
A. The patient may cough a lot due to increased amounts of bronchial secretions.
B. Increased need to urinate due to renal vasodilation.
C. Higher heart rate.
D. Watery saliva.
E. Depressed blood pressure.
Answer 2
A. The patient may cough a lot due to increased amounts of bronchial secretions. – Secretions are often the result of parasympathetic activity. We are expecting sympathetic effects because pheochromocytoma is a tumor on the adrenal medulla, leading to systemic sympathetic effects.
B. Increased need to urinate due to renal vasodilation. – This is also a parasympathetic response; urination is often inhibited during a sympathetic response as this would be unhelpful during the fight or flight response.
C. Higher heart rate. – CORRECT. This would result from a sympathetic stimulus and is the correct answer.
D. Watery saliva. – This results from parasympathetic activity, not sympathetic.
E. Depressed blood pressure. – A higher than normal blood pressure would actually be expected based on the sympathetic effects caused by pheochromocytoma.

Great question
You are in an acute care hospital setting seeing a 45-year-old male patient for the first time since his surgery three days ago. He has a past medical history of diabetes and syphilis. After performing all necessary safety checks, you assess his vitals and prepare to mobilize him. When he stands up from the bed, he reports feeling very lightheaded. You believe he is experiencing orthostatic hypotension. Being a UDPT student, you immediately assess his vitals. What do you expect to see when assessing his blood pressure and heart rate?
A. Drop of diastolic blood pressure greater than 20 mmHg and increased heart rate greater than 10 beats per minute.
B. Drop in systolic blood pressure greater than 20 mmHg and an increase in heart rate greater than 10 beats per minute.
C. Drop in systolic blood pressure greater than 20 mmHg and a decrease in heart rate greater than 10 beats per min.
D. Rise in systolic blood pressure greater than 20 mmHg and an increase in heart rate greater than 10 beats per minute.
E. Drop of diastolic blood pressure greater than 20 mmHg and a decrease heart rate greater than 10 beats per minute.
Answer 3
A. Drop of diastolic blood pressure greater than 20 mmHg and increased heart rate greater than 10 beats per minute. - Drop in diastolic blood pressure greater than 20 mmHg is not expected with orthostatic hypotension.
B. Drop in systolic blood pressure greater than 20 mmHg and an increase in heart rate greater than 10 beats per minute –CORRECT. Orthostatic hypotension is diagnosed by these symptoms.
C. Drop in systolic blood pressure greater than 20 mmHg and a decrease in heart rate greater than 10 beats per min. – A decrease in heart rate is not a symptom of orthostatic hypotension.
D. Rise in systolic blood pressure greater than 20 mmHg and an increase in heart rate greater than 10 beats per minute. –A rise in systolic blood pressure is not a symptom of orthostatic hypotension.
E. Drop of diastolic blood pressure greater than 20 mmHg and a decrease heart rate greater than 10 beats per minute. - Neither a drop in diastolic or in heart rate are symptoms of orthostatic hypotension.

Excellent
You are examining a sixteen-year-old patient with a torn ACL. You are wearing latex gloves when you begin to do a Lachman’s test. The patient complains that she feels like her throat is closing up, so you ask her if she is allergic to latex and she replies yes. She then pulls out her epi-pen and injects herself. What responses do you expect the epi-pen to induce?
A. Bronchoconstriction
B. Increase in watery salivary secretion
C. Increase in viscous salivary secretion
D. Vasodilation
E. A tiger to start chasing her
Answer 4
A. Bronchoconstriction- Bronchoconstriction is a parasympathetic response. Bronchodilation is a sympathetic response.
B. Increase in watery salivary secretion- this response is a parasympathetic response.
C. Increase in viscous salivary secretion- CORRECT! An increase in viscous salivary secretion is a sympathetic response.
D. Vasodilation- vasoconstriction is the general response to epinephrine
E. A tiger to start chasing her- although a tiger chasing you might cause an increase in epinephrine; an increase in epinephrine will not cause a tiger to chase you.

Good
You are working in an outpatient orthopedic clinic trying to teach a 17-year-old track athlete to become a forefoot runner. You put the patient on the treadmill and instruct her to run using the forefoot gait for 10 minutes. After 5 minutes your patient stops and says she needs to take a break because her stomach is cramping. When you ask her if she just ate, she responds, "Yes, I ate 2 slices of Grottos pizza on my walk over here.” Inhibition to which plexus would cause a disruption in peristalsis?
A. Meissner's plexus
B. Submucosal plexus
C. Myenteric plexus
D. Brachial plexus
E. Lumbar plexus
Answer 5
A. Meissner's plexus - This cause secretions in the gut
B. Submucosal plexus - This is the same as A
C. Myenteric plexus – CORRECT. Exercise causes sympathetic discharge, which would inhibit the myenteric plexus. Inhibition to this plexus causes peristalsis to come to a halt - leading to cramps.
D. Brachial plexus - This has nothing to do with the enteric division of the autonomic nervous system
E. Lumbar plexus - This has nothing to do with the enteric division of the autonomic nervous system

Might be too easy… but good question
Upon initial evaluation your patient tells you that she has frequent panic attacks. She is afraid to tell you because sometimes she feels like she is just crazy. You explain to her that these panic attacks are just an overactive discharge of her _________ nervous system and that she is not crazy. You also remember that this system is sometimes referred to as the _________ system, named after the location of the preganglionic cell bodies.
A. Parasympathetic, craniosacral
B. Sympathetic, lumbosacral
C. Parasympathetic, thoracolumbar
D. Sympathetic, thoracolumbar
E. Enteric, craniosacral
Answer 6
A. Parasympathetic, craniosacral - these 2 go together, but panic disorder is an overactive discharge of the sympathetic nervous system.
B. Sympathetic, lumbosacral - the system is correct, but the sympathetic nervous system has preganglionic cells in the thoracolumbar region not lumbosacral.
C. Parasympathetic, thoracolumbar - Thoracolumbar is correct, but Panic disorder causes a sympathetic discharge not parasympathetic.
D. Sympathetic, thoracolumbar – CORRECT. - Panic disorder causes a sympathetic discharge and the preganglionic cell bodies are in the thoracolumbar region.
E. Enteric, craniosacral - wrong system and wrong location.

Excellent question
A 24-year-old graduate student comes to your clinic because he wants to lose weight. As part of developing a healthy weight-loss program for him, you decide to do a 1-Mile-Run Test to establish his baseline fitness and estimate his VO2 Max. After the test, he asks you why his saliva seemed to get thicker when he was in the middle of his run. You explain that:
A. The sympathetic nervous system causes salivary secretions to thicken when it is activated.
B. The parasympathetic nervous system causes salivary secretions to thicken when it is activated.
C. Increased respiration rate causes the mucosal membranes in the mouth and sinuses to dry out, leading to thicker mucous.
D. Increased respiration rate causes irritation of the primary and secondary bronchi, which leads to thickened mucous production as a protective agent.
E. It’s probably just his imagination.
Answer 7
A. The sympathetic nervous system causes salivary secretions to thicken when it is activated. – CORRECT. This may be because it would not be helpful to salivate during high stress scenarios, whereas watery saliva would be helpful during eating (parasympathetic) to allow for enzymes in the saliva to adequately assist with digestion.
B. The parasympathetic nervous system causes salivary secretions to thicken when it is activated. - Incorrect, the parasympathetic nervous system causes saliva to become more watery.
C. Increased respiration rate causes the mucosal membranes in the mouth and sinuses to dry out, leading to thicker mucous. – The sympathetic nervous system effects should play a much larger role in causing the thicker saliva than increased respiration itself.
D. Increased respiration rate causes irritation of the primary and secondary bronchi, which leads to thickened mucous production as a protective agent. - While irritation of the upper-respiratory system may lead to increased mucous production, this is not the primary factor at work in this instance.
E. It’s probably just his imagination. – Thickened saliva is a normal response of the sympathetic nervous system, so the patient is noticing something that normally occurs in his body.

I like the question a lot, but I didn’t really emphasize this too much in class. I might reinforce it for the final.
Your patient is a 32-year-old woman training for a marathon. She is experiencing pain in her knee secondary to patellofemoral pain syndrome and you want to complete a full running evaluation on her. In order for you to complete your exam you need to see her run, so you set her up on the treadmill. A few minutes into the run she becomes short of breath and starts to wheeze. She immediately gets off the treadmill to take her short acting β2 agonist. Which of the following effects would NOT occur?
A. Increase in heart rate
B. Increase in contractile force
C. Bronchodilation
D. Watery saliva
E. Decreased motility
Answer 8
A. Increase in heart rate- The heart has β2 and β1 receptors and that would cause in increase in heart rate to occur.
B. Increase in contractile force- The heart has β2 and β1 receptors and that would cause in increase in heart rate to occur.
C. Bronchodilation- The airways of the lungs have β2 receptors that would cause bronchodilation to occur.
D. Watery saliva- CORRECT. The glands have β2 and ά1 receptors that would cause an increase in viscous saliva.
E. Decreased motility- The stomach has β2 and ά1 receptors that would cause decreased motility.

Ok, you guys are focused too much on the saliva! I’ll probably re-word this question a little.
You have a 20-year-old female field hockey player with ITB pain referred to you for gait analysis. After 10 minutes of jogging on the treadmill, you notice she is beginning to sweat. Postganglionic fibers release which neurotransmitter to cause this response?
A. Acetylcholine
B. Norepinephrine
C. Substance P
D. Dopamine
E. GABA
Answer 9
A. Acetylcholine – CORRECT.- Normally sympathetic postganglionic fibers release Norepinephrine EXCEPT sweat glands, they release Acetylcholine.
B. Norepinephrine - Most sympathetic postganglionic fibers release norepinephrine, however sweat glands are the only exception and they release Acetylcholine.
C. Substance P – This is used in the myenteric plexus of the enteric division of the autonomic nervous system, not the sympathetic division.
D. Dopamine – This plays a role in the basal ganglia and Parkinson’s disease, but would not be a primary neurotransmitter released by the sympathetic nervous system.
E. GABA – Although this is an important neurotransmitter within the brain, GABA is not a primary neurotransmitter for the sympathetic nervous system

Good question
You are on your acute care affiliation and you meet with the nurse prior to entering your first patient’s room. She informs you that your patient has been on bed rest for 5 days and just finished with lunch. The nurse noted that the patient has pupillary constriction, increased mucous production in her bronchioles, and has been experiencing bronchoconstriction. When you enter the room, the patient is asleep with watery drool and has a heart rate of 55 bpm. You remember from neuroanatomy that all of these symptoms are coming from the same division of the autonomic nervous system. Which of the following cranial nerves is not involved in these responses from this system?
A. CN III
B. CN VII
C. CN IX
D. CN X
E. CN XI
Answer 10
A. CN III - All of these symptoms are coming from the parasympathetic system (rest and digest). This division has outflow from cranial nerves 3, 7, 9, 10.
B. CN VII - All of these symptoms are coming from the parasympathetic system (rest and digest). This division has outflow from cranial nerves 3, 7, 9, 10.
C. CN IX - All of these symptoms are coming from the parasympathetic system (rest and digest). This division has outflow from cranial nerves 3, 7, 9, 10.
D. CN X - All of these symptoms are coming from the parasympathetic system (rest and digest). This division has outflow from cranial nerves 3, 7, 9, 10.
E. CN XI - CORRECT - The spinal accessory nerve does not come from the parasympathetic nervous system.

Good question – I might change it to a different CN.
A 4 year old boy who came into the pediatric outpatient clinic has a diagnosis of spina bifida. The mother tells you that she did not know she was pregnant until 4 months and was not taking folic acid.
Question 1:
What germ layer is affected in Spina Bifida?
a. Ectoderm
b. Mesoderm
c. Somite
d. Endoderm
e. Notochord
Answer 1:
a. Ectoderm- CORRECT- The Spinal Cord is derived here
b. Mesoderm- This layer develops into the Notochord and then becomes the somites
c. Somite- This comes from the Mesoderm and develops into the musculoskeletal system
d. Endoderm- This layer becomes the lining of the hollow organs
e. Notochord- This comes from the Mesoderm and has residual existence in the nucleus pulposus
Question 2:
What structure of the previous germ layer causes Spina Bifida when it does not fully close?
a. Neural Crest Cells
b. Somites
c. Neural tube
d. Neural Plate
e. Notochord
Answer 2:
a. Neural Crest Cells- This develops into the PNS
b. Somites- This develops into the musculoskeletal system
c. Neural tube- CORRECT- this develops into the spinal cord
d. Neural Plate- This includes all three germ layers
e. Notochord- This comes from the Mesoderm and has residual existence in the nucleus pulposus
A 17 y.o. pt with Addison’s Disease who has a severe deficiency in epinephrine and norepinephrine comes into the orthopedic outpatient clinic complaining of chronic fatigue and low back pain due to spasm and weakness.
Question 3:
These hormones are secreted by what gland?
a. Pituitary
b. Lacrimal
c. Mammillary
d. Adrenal Medulla
e. Thyroid
Answer 3:
a. Pituitary- Growth hormone and ACTH production
b. Lacrimal- Tear production
c. Mammillary- Lactate production
d. Adrenal Medulla- CORRECT- release epinephrine and norepinephrine
e. Thyroid- Stimulating hormone production
A 30 y.o. pt who is in the hospital s/p a fall has loss of sensation in the C6 and C7 dermatomal regions.
Question 4:
From what germ layer and structure do these dermatomes arise from?
a. Endoderm and Neural Crest Cell
b. Mesoderm and Somite
c. Ectoderm and Neural Crest Cell
d. Mesoderm and Neural Crest Cell
e. Ectoderm and Somite
Answer 4:
a. Endoderm and Neural Crest Cell- The endoderm develops into the lining of the hollow organs and the neural crest cell develops into PNS
b. Mesoderm and Somite- CORRECT- The mesoderm develops into the notochord and then into the somites which give rise to the dermatomes
c. Ectoderm and Neural Crest Cell- Ectoderm develops into the Spinal Cord and Nueral Crest Cell develops into the PNS
d. Mesoderm and Neural Crest Cell- Although the mesoderm is the correct germ layer, the neural crest develops into the PNS, not the dermatomes
e. Ectoderm and Somite- Although the Somite does give rise to the dermatomes, the ectoderm is not the germ layer that gives rise to the somites
A 45 y.o. pt presents to your outpatient clinic with a diagnosis of MULTIPLE SCLEROSIS (MS). They are presenting with loss of balance and difficulty walking. MS affects the white mater of the spinal cord.
Question 5:
What layer develops into white mater during radial migration?
a. Mental layer
b. Ventricular layer
c. Subventricular layer
d. Marginal layer
e. Weiss layer
Answer 5:
a. Mental layer- this develops into the gray mater
b. Ventricular layer- becomes ependymal layer
c. Subventricular layer- temporary and only exists in the cerebellum and cerebrum
d. Marginal layer- CORRECT- this develops into the white mater
e. Weiss layer- We made this layer up with the inspiration of our TOTALLY AWESOME neuroanatomy professor
A pt presents to your outpatient rehabilitation facility s/p shoulder relocation. He has weakness in wrist extension (radial nerve distribution).
Question 6:
FROM what cells do peripheral nerves like the radial nerve develop?
a. Ependymal
b. Oligodendrocytes
c. Astrocytes
d. Schwann Cells
e. Neural Crest Cells
Answer 6:
a. Ependymal- interlining of ventricles
b. Oligodendrocytes- myelination of CNS
c. Astrocytes- support structure, blood brain barrier
d. Schwann Cells- myelination of PNS
e. Neural Crest Cells- CORRECT- develop into the PNS
You are treating a pregnant woman who just had an WHOSE ultrasound that RECENTLY indicated her baby has anencephaly, an absence of a large part of the brain and the skull. This is one of the most common neural tube defects.
Question 7:
What germ layer is the neural tube derived from?
a. Endoderm
b. Mesoderm
c. Ectoderm
d. Myoderm
e. Escoderm
Answer 7:
a. Endoderm- This develops into the lining of the hollow organs
b. Mesoderm- This develops into the notochord and then the somites
c. Ectoderm- CORRECT- the neural tube comes from the neural groove and fold which orginates from the ectoderm
d. Myoderm- We made this up, this is not a true germ layer
e. Escoderm- We made this up, this is not a true germ layer
What nutritional supplements should be added to her diet? AFTER THE FACT? MAYBE REFRAME THE QUESTION
a. Omega-3
b. Folate
c. Vitamin C
d. Vitamin D
e. Iron
Answer 8:
a. Omega-3- This was not one of the vitamins listed to provide prenatal benefits for the growth of the neural plate
b. Folate – CORRECT – This vitamin can help the prenatal development of the fetus’ neural plate from approximately 13-19 days
c. Vitamin C- This was not one of the vitamins listed to provide prenatal benefits for the growth of the neural plate
d. Vitamin D- This was not one of the vitamins listed to provide prenatal benefits for the growth of the neural plate
e. Iron- This was not one of the vitamins listed to provide prenatal benefits for the growth of the neural plate
You are seeing a 13 y.o. boy who has chronic sprained ankles. Upon reviewing his medical history you notice he has a history of epileptic seizures, a developmental neural deficit partially due to abnormal migration.
Question 9:
What are the 3 layers of radial migration in proper order from inside to out?
a. Ventricular, Marginal, Mantel
b. Ventricular, Mantel, Marginal
c. Mantel, Ventricular, Marginal
d. Marginal, Ventricular, Mantel
e. Mantel, Marginal, Ventricular
Answer 9:
a. Ventricular, Marginal, Mantel- Not the correct order
b. Ventricular, Mantel, Marginal- CORRECT- This is the proper order of radial migration from inside to out
c. Mantel, Ventricular, Marginal- Not the correct order
d. Marginal, Ventricular, Mantel- Not the correct order
e. Mantel, Marginal, Ventricular- Not the correct order
Case 8: You are treating a pt in the Sports and Ortho Clinic at the University of Delaware for Radiculopathy caused by a bulging disc.
Question 10:
What part of the neural tube is left as a remnant in the nucleus pulposus of the vertebral disc and what germ layer did it arise from?
a. Somite and Mesoderm
b. Neural Groove and Ectoderm
c. Notochord and Endoderm
d. Somite and Ectoderm
e. Notochord and Mesoderm
Answer 10:
a. Somite and Mesoderm- This pair matches correctly but this eventually develops into the dermatomes, sclerotomes, and myotomes
b. Neural Groove and Ectoderm- This pair matches correctly but this eventually develops into the spinal cord
c. Notochord and Endoderm- The notochord is correct but it does not come from the endoderm
d. Somite and Ectoderm- This pair does not match up properly
e. Notochord and Mesoderm- CORRECT- What is left of the notochord exists in the nucleus pulposus and arises from the mesoderm
Case 1:
You are treating a 50 y.o. patient in the acute care setting. You notice pseudoptosis, miosis, and anhydrosis so you suspect that the patient may have Horner’s Syndrome.
Question 1:
What is the importance of recognizing the signs and symptoms of Horner’s Syndrome?
a. Possibility of a Seizure – ARE YOU SURE THIS ISN’T LINKED TO PARTIAL SEIZURES???
b. Autonomic Dysreflexia
c. High Blood Pressure
d. Pancoast Tumor
e. Overactive discharge of the sympathetic nervous system
Answer 1:
a. Possibility of a Seizure- Seizures are not linked to Horner’s Syndrome
b. Autonomic Dysreflexia- this is due to noxious stimulus
c. High Blood Pressure- this is linked to pheochromocytoma
d. Pancoast Tumor – CORRECT – These symptoms together could be warning signs of a pancoast tumor (a tumor on the apex of the lung)
e. Overactive discharge of the sympathetic nervous system- This is linked to anxiety/panic disorder
Case 2:
You are treating a 34 y.o. boxer in an acute care setting that just finished a boxing match. He suffered a severe blow to the head. He is having increased floaters and feelings of heaviness in his eye.
Question 2:
What structure in his eye do you believe might be detached?
a. Retina
b. Lens
c. Optic Nerve
d. Sclera
e. Anterior Chamber
Answer 2:
a. Retina – CORRECT – The retina is not physically attached to anything keeping it in place and can easily be detached with a hard enough blow to the head
b. Lens- the lens lies in the posterior chamber with the iris and is not easily detached – NOT TRUE
c. Optic Nerve- The optic nerve has many connections and is not easily detached
d. Sclera- this is the white of the eye and is continuous with the cornea
e. Anterior Chamber- the anterior chamber consists of the lens and cornea which have many connections and are not easily detached
You are treating a 25 y.o. female in an acute care setting with a T8 spinal cord injury who you must teach how to perform transfers. Patients with T8 tend to be at risk for Autonomic Dysreflexia.

Question 3:

During Automatic Dysreflexia, what response do you see with HR and BP?

a. HR increases, BP decreases
b. HR decreases, BP increases
c. HR increases, BP increases
d. HR decreases, BP decreases
e. HR stays the same, BP stays the same
Answer 3:

a. HR increases, BP decreases- the opposite occurs
b. HR decreases, BP increases – CORRECT- this is what we see in autonomic dysreflexia
c. HR increases, BP increases- HR actually decreases
d. HR decreases, BP decreases- BP actually increases
e. HR stays the same, BP stays the same- HR actually decreases and BP increases
A 17 y.o. female patient comes into the clinic post-op ACL reconstruction. During treatment, she seems excessively worried and is sweating and shaking. She says she suffers from frequent upset stomachs and insomnia. You believe that she is suffering from an Anxiety or Panic Disorder.

Question 4:

Her symptoms are caused by an increase in which division of the nervous system?

a. Sympathetic
b. Enteric
c. Parasympathetic
d. Sensory
e. Somatic
Answer 4:

a. Sympathetic – CORRECT – there is an overactive discharge of the sympathetic nervous system in anxiety/panic disorders
b. Enteric- there would be decreased peristalsis with anxiety causing the upset stomachs
c. Parasympathetic- the symptoms would be opposite with a parasympathetic response
d. Sensory- anxiety disorder does not effect the sensory system
e. Somatic- the somatic system is not included in anxiety disorder
An 80 y.o. female patient comes into the clinic for treatment of low back pain. While treating her she mentions that she has cloudy eye vision and sudden severe eye pain and feels like she has increased pressure in her eye. – PATIENTS DON’T COMPLAIN OF INCREASED PRESSURE… IT IS MEASURED BY THE OPHTHALMOLOGIST
Question 5:
Hearing this you believe that she might have what eye disease?
a. Detached Retina
b. Homonymous Hemianopsia
c. Pie in the Sky Lesion
d. Glaucoma
e. Bitemporal hemianopsia
Answer 5:
a. Detached Retina – increased floaters
b. Homonymous Hemianopsia – only seeing half of the visual field
c. Pie in the Sky Lesion – only able to see upper ¼ of the visual field
d. Glaucoma – CORRECT – increase pressure in the eye that can lead to blindness
e. Bitemporal hemianopsia- tunnel vision, loss of peripheral vision due to a cut/blockage LESION at the CENTRAL optic chiasm
You are treating a 50 year old female in an acute care setting. She recently suffered a stroke to part of her internal carotid artery and is having trouble with her vision.
Question 6:
Which artery do you think was affected during her stroke?
a. Anterior Cerebral Artery
b. Posterior Communicating Artery
c. Facial Artery
d. Maxillary Artery
e. Ophthalmic Artery
Answer 6:
a. Anterior Cerebral Artery – supplies frontal lobe and parietal lobe
b. Posterior Communicating Artery – connection artery in Circle of Willis
c. Facial Artery – branch of external carotid artery
d. Maxillary Artery – branch of external carotid artery
e. Ophthalmic Artery – CORRECT – blood supply to the retina, 1st branch of internal carotid artery
An 88 y.o. female you are seeing in your outpatient clinic fell while she was walking to the bathroom at night. She reports not turning on the light because it is only a short distance to her bathroom.
Question 7:
All of the following are characteristics of night vision except: ARE THESE CHARACTERISTICS OF NIGHT VISION OR ROD PHOTORECEPTORS?
a. High sensitivity to light
b. More abundant photosensitive pigment
c. Slow response to light
d. Low sensitivity to light
e. Occurs in the rods
Answer 7:
a. High sensitivity to light – occurs in rods
b. More abundant photosensitive pigment – occurs in rods
c. Slow response to light – occurs in rods
d. Low sensitivity to light – CORRECT – occurs in cones
e. Occurs in the rods – occurs in rods
You are treating a patient in the acute care setting post MVA who is experiencing decreased visual acuity. This happens as a result to damage of cone photoreceptors in the retina.
Question 8:
All of the following are true of cones except:
a. Mediate color vision
b. Saturate in daylight
c. Less numerous than rods
d. High concentration in fovea
e. Fast response to light
Answer 8:
a. Mediate color vision – occurs in cones
b. Saturate in daylight – CORRECT – occurs in rods
c. Less numerous than rods – occurs in cones
d. High concentration in fovea – occurs in cones
e. Fast response to light – occurs in cones
You are seeing an infant in the acute care setting for abnormal reflexes. You notice on their medical history that they were diagnosed with Hirschprung’s Disease. In this disease, peristalsis does not occur due to agenesis of the myenteric plexus IN THE DISTAL COLON. – GRAMMAR FROM SINGLE TO PLEURAL
Question 9:
What portion of the nervous system causes increased peristalsis?
a. Somatic Nervous System
b. Autonomic Nervous System
c. Peripheral Nervous System
d. Central Nervous System
e. Sensory (Afferent) Division
Answer 9: THE ANS IS PART OF THE PERIPHERAL NERVOUS SYSTEM
a. Somatic Nervous System- Not connected to peristalsis
b. Autonomic Nervous System – CORRECT – peristalsis is controlled by the sympathetic and parasympathetic systems
c. Peripheral Nervous System- Not connected to peristalsis
d. Central Nervous System- Not connected to peristalsis
e. Sensory (Afferent) Division- Not connected to peristalsis
What sign should parents look for as a good sign that their infant does NOT have Hirschprung’s Disease?
a. Crying
b. Eating
c. Pooping
d. Sleeping
e. Laughing
Answer 10:
a. Crying- this reflects strong lungs
b. Eating- not related to this disease
c. Pooping- CORRECT- this is a sign that peristalsis is occurring
d. Sleeping- not related to this disease
e. Laughing- this is just cute to see, but not related
1. On your acute care affiliation you see your patient a 42 year old African American female. Upon review of her charts, you see her most recent MRI films. It shows a bubble shaped bleed. What layer of the brain is the hematoma most likely to be located?

a. Subarachnoid Space
b. Epidural Space
c. Falx Cerebri
d. Subdural Space
e. Pia Space
a. Subarachnoid Space (Incorrect - subarachnoid space acts as more of a drainage area so bleeds would not bubble out)
b. Epidural Space (Correct Answer - the suture lines of the skull act to tack this layer down which forms the bubble)
c. Falx Cerebri (Incorrect - a part of the inner layer that divides left side of the brain from the right)
d. Subdural Space (Incorrect - the layer is loose and the blood is not confined to as small an area)
e. Pia Space (Incorrect - does not exist)

I will probably include a variant of this question but will word it differently. Bubble-shaped is not typically an acceptable term, although I use it as an analogy for an epidural bleed. The correct description is a “convex” shaped bleed. Also, you ask “what layer” of the brain, however, the question really should ask, what meningeal space, or space within the cranium… the epidural space is not part of the brain proper.
2. Your patient is a 23 year old male in the ICU. He was admitted after a car accident in which he sustained a TBI. You are evaluating for strength deficits, proprioception and sensation. You check his chart before you knock and notice a physician’s note telling all health personnel to be alert for signs and symptoms of (an expanding) subdural hematoma. Where does the subdural hematoma occur?

a. Between the arachnoid mater and the dura mater
b. Between the bone and the dura mater
c. Between the arachnoid mater and the pia mater
d. Between the pia mater and the brain
e. Between the sulci and the gyri
a. Between the arachnoid mater and the dura mater. (Correct - this is where it occurs)
b. Between the bone and the dura mater. (Incorrect - this is known as an epidural hematoma space?)
c. Between the arachnoid mater and the pia mater. (Incorrect - this is known as a subarachnoid bleed space?)
d. Between the pia mater and the brain. (Incorrect - this means there will be a bleed into the brain) aka intraparenchymal extension
e. Between the sulci and the gyri (Incorrect - this answer choice doesn’t make sense) – ACTUALLY THIS WOULD ALSO BE CONSIDERED SUBARACHNOID SPACE ON CT AND MRI.

Be careful with wording of explanatons.
3. A 26 year old male is on your schedule in your acute care setting. You have not seen your patient but you have access to their HIS MRI. Upon observation of the MRI you notice blood in the sulcis (THIS IS NOT A WORD) of the brain. This means they have had a bleed in what layer of the brain?

a. Subdural space
b. Epidural space
c. Subarachnoid space
d. Epiarachnoid space
e. None of the above
a. Subdural space (incorrect- the blood will not go into the sulci)
b. Epidural space (incorrect- the is the most superficial space and there are both the dura, and arachnoid mater to pass through before reaching the sulci)
c. Subarachnoid space (correct- blood here can reach into the sulci of the brain)
d. Epiarachnoid space (incorrect- this does not exist)
e. None of the above (incorrect, it is in the subarachnoid space)
4. You are working in the acute care setting and your patient, a 32 year old male, is not producing enough CSF. Which cells in his body may not be operating properly?

a. Microglia
b. Cells of the choroid plexus
c. Hypodymalcells
d. Cells of the glial limitans
e. Schwann cells

I don’t know if this clinical scenario exists, or if it does, I have yet to see it. I will reword the question to include something related to the drainage of CSF being inadequate.
a. Microglia (incorrect- these cells participate in the inflammatory and degenerative reactions in neurons)
b. Cells of the choroid plexus (correct- these cells are responsible for filtering the blood to create CSF)
c. Hypodymalcells (incorrect- these cells do not exist)
d. Cells of the glial limitans (incorrect- though these cells are involved in the separating the brain from the ventricular CSF, they are not involved in creating CSF)
e. Schwann cells (incorrect- these cells are responsible for making the myelin in the PNS)
5. You are treating a 35 year old woman in an outpatient setting. She has been taking lithium to treat her depression for many years. She has recently found out she is pregnant and asks for your advice regarding her medications. As a PT student, you know that lithium could be harmful to a fetus. What do you do?

a. Do not worry about it. If it is a safe dose for her it should be safe for the baby
b. Stop taking her medication immediately
c. Tell her to taper her dose off over the next two weeks to avoid withdrawal and protect her developing baby from long term exposure to the drug
d. Refer to a physician who will take her personal medical history into account to determine what the safest option is for both her and her unborn baby
e. None of the above
a. Do not worry about it. If it is a safe dose for her it should be safe for the baby. (incorrect- something that is safe for an adult is not necessarily safe for a baby, let alone a developing fetus, the fetus’s BBB is not mature at this point)
b. Stop taking her medication immediately (incorrect- the side effects she could experience could be detrimental to the developing fetus)
c. Tell her to taper her dose off over the next two weeks to avoid withdrawal and protect her developing baby from long term exposure to the drug. (incorrect- it is out of our practice to make any recommendations like this)
d. Refer to a physician who will take her personal medical history into account to determine what the safest option is for both her and her unborn baby. (correct- always refer to a physician in regards to their stopping or starting prescription medication use)
e. None of the above (incorrect- you should refer her to the proper medical professionals)

This is a professional issue related question rather than a basic neuroscience question. I might reword it to test the concept of teratogenicity of what we discussed in class.
6. Jake is an 18 y.o freshman at the University of Delaware, and has recently been admitted to the hospital because of bacterial meningitis. You are an inpatient PT at the hospital and strict infectious control measures have been taken. You know it is important to follow these measures because bacterial meningitis compromises the blood brain barrier. What is the blood brain barrier comprised of?

a. Schwann cells
b. Oligodendrites
c. Sarcomeres
d. astrocytes
e. Neurons
a. Schwann cells (incorrect this is the myelin of the PNS)
b. Oligodentrites (incorrect these surround the axons of the CNS).
c. Sarcomeres (incorrect these are muscle cells)
d. astrocytes (correct these cells make up the blood brain barrier)
e. Neurons (incorrect these cells transport action potentials)

Good question. I will make some of the answer choices more challenging.
7. You are working on a trauma unit and you have a 45 y.o. female named Barbara who was hit in the head by a foul ball at a baseball game. Barbara has imaging done and it shows that she is bleeding into the subarachnoid space. What is your main concern?

a. This is normal. This is the way blood is drained into the venous system.
b. IMMEDIATE Brain herniation and death can be the result of this type of bleeding.
c. Hydrocephalus can occur with this type of bleeding.
d. Your patient is having a thromboses in the venous system of the brain.
e. Your patient could bleed out and a blood transfusion may be needed.
a. This is normal. This is the way blood is drained into the venous system. (incorrect - CSF is drained from the subarachnoid space)
b. Brain herniation and death can be the result of this type of bleeding. (incorrect - this is bleeding in the epidural space)
c. Hydrocephalus can occur with this type of bleeding. (correct - clots in the arachnoid granulations cause back up of CSF)
d. Your patient is having a thromboses in the venous system of the brain. (incorrect - the subarachnoid space is not part of the venous system of the brain. It is part of the meninges)
e. Your patient could bleed out and a blood transfusion may be needed. (incorrect - bleeding in the subarachnoid space (ALONE) does not cause this)

Subarachnoid blood can cause hydrocephalus which can eventually lead to herniation.
8. You are working in an acute care setting and your 20 y.o. patient was in a car accident. As a consequence of the accident, the system that maintains homeostasis of her nervous system (TISSUE) was disrupted. What system maintains homeostasis of the nervous system?

a. Blood:CSF barrier
b. Blood brain barrier
c. CSF: Brain barrier
d. Vestibular system
e. Cardiovascular system
a. Blood:CSF barrier (incorrect - keeps blood and CSF from mixing)
b. Blood brain barrier (correct - maintains homeostatic environment by preventing free exchange of fluid)
c. CSF: Brain barrier (incorrect - separates CSF from brain tissue)
d. Vestibular system (incorrect - helps with balance)
e. Cardiovascular system (incorrect - this system maintains gas exchange)

I’m going to change the wording a little on this one. Technically, all three barriers are involved in maintaining homeostasis. The one that is most directly related to neuronal tissue is the BBB.
9. Whitney Bobboski is a 45 year old female. She is a diabetic and forgot to consume sugars before starting her journey across the country. She fainted and got into a car accident. She was admitted into the ICU and had a CT scan and MRI of her head. The imaging showed that there was some bleeding in the subdural space. Because you are a DPT graduate from the University of Delaware, you know that her condition is serious and the blood can travel the entire half of the brain, but you also know that the blood can’t travel to the other hemisphere. What is the structure that prevents the blood from traveling to the other hemisphere?

a. Ependymal cells
b. Choroid plexus
c. Falx cerebri
d. Crista galli
e. Ventricular septum
a. Ependymal cells (incorrect - these cells separate the CSF in the ventricles from the brain tissue.)
b. Choroid plexus (incorrect - this filters the blood to create CSF)
c. Falx cerebri (correct - this structure is located between the two hemispheres)
d. Crista galli (incorrect - this is a bony process where the falx cerebri attaches to anteriorly)
e. Ventricular septum (incorrect - this divides the two ventricles of the human heart)

Good question. I will only change the wording around slightly. (kind of lengthy!)
10. Jessica Zow is a 22 year old female, that came into your outpatient PT clinic after sustaining an open tibial fracture secondary to falling off of her bike. When looking at the EMT record, you notice that Jessica suffered from primary shock, most likely from the pain of the fracture and the stress of the trauma. As a PT you know that primary shock causes a decrease in peripheral blood flow and cardiac output. What is/are the mechanism/mechanisms that the body uses to ensure that blood flow and blood pressure in the brain is consistent?

a. Cerebral Vessel responses due to circulating metabolites
b. Auto-regulation through compensatory changes in the diameter of cerebral arterioles
c. Sympathetic innervation of the cerebral vessels
d. A and B
e. All of the above
a. Cerebral Vessel responses due to circulating metabolites (Incorrect) Although Cerebral Vessels respond to circulating metabolites such as CO2, O2 and pH, this is not the only correct answer.
b. Auto-regulation through compensatory changes in the diameter of cerebral arterioles (Incorrect) Cerebral arterioles use auto-regulation to either constrict or relax in order to keep a consistent blood pressure or flow to the brain, however this is not the only correct answer.
c. Sympathetic innervation of the cerebral vessels (Incorrect) Sympathetic innervation to the cerebral blood vessels plays a minor role in the regulation of cerebral blood flow, however it is not the only correct answer.
d. A and B (Incorrect) A and B are correct however C is also correct.
e. All of the above (Correct) All of the answers listed above are how cerebral blood
flow is regulated.

I should have mentioned I don’t use A and B or all of the above answer choices. I’ll likely modify this question significantly.
11. Randy Jabinski is a 32 year old UDPT student currently being treated for rhabdomyolysis secondary to an increase in training intensity for his upcoming 6 minute walk test. His significant past medical history includes Insulin Dependent Diabetes Mellitus. While reviewing his chart you notice that Randy is noted for noncompliance with regulation of his insulin medications. Is regulating insulin important for the nervous system?

a. Yes. A constant source of glucose is required by nervous tissue for proper functioning.
b. No. Cells in the nervous system are majorly anaerobic and are therefore unaffected by insulin.
c. Yes. Nervous tissue contain actin and myosin which require appropriate levels of glucose for proper functioning.
d. No. Mr. Jabinksi is a PT student and doesn’t have time to be worrying about things that don’t make a significant difference in the body.
e. None of the above.
a. Yes. A constant source of glucose is required by nervous tissue for proper functioning. (Correct)
b. No. Cells in the nervous system are majorly anaerobic and are therefore unaffected by insulin. (Incorrect) Cells in the nervous system, with the exception of some glia cells, do not have an anaerobic source of energy.
c. Yes. Nervous tissue contain actin and myosin which require appropriate levels of glucose for proper functioning. (Incorrect) Nervous tissue do not contain actin and myosin. Actin and myosin are only present in skeletal muscle.
d. No. Mr. Jabinksi is a PT student and doesn’t have time to be worrying about things that don’t make a significant difference in the body. (Incorrect) Insulin is necessary in the regulation of glucose, which is extremely important in keeping the brain and nervous tissue functioning.
e. None of the above. (Incorrect) Letter A is the correct answer.

I like that you included Diabetes but the main nervous system considerations for diabetics include peripheral neuropathy, autonomic neuropathies, stocking glove patterns, etc. You could have given choices about the blood supply to a peripheral nerve in addition to metabolic requirements. This question, unfortunately, is not written at a graduate student level. I will not use it.
1. You are in an outpatient clinic and treating a 56 year old female, who complains of frequently losing her balance (GRAMMAR). During your initial evaluation you notice that she has problems with her vision (WHAT KIND OF PROBLEMS), which could be the cause of her LOB. She is currently in remission of brain cancer. Given the loss of balance and her PMH, where might you suspect this patient to have a lesion?

a. Pituitary gland
b. Pineal gland
c. Parotid gland
d. Thyroid gland
e. Mammillary gland
a. Pituitary gland (Correct - the pituitary gland is located right near the optic chiasm and could directly effect vision)
b. Pineal gland (Incorrect - main melatonin producer not located near optic nerves/tracts)
c. Parotid gland (Incorrect - this is a salivary gland located on the jaw line)
d. Thyroid gland (Incorrect - this gland is located in the neck and typically associated with weight loss, heart rate/blood pressure, goiter, etc)
e. Mammillary gland (Incorrect - mammillary glands are not located in the brain)

You should test (or at least incorporate) the nervous system concept related to the visual pathway… not just the proximity of the glad to the visual system. What kind of visual field deficit might this patient have?
2. You are in an inpatient setting treating a middle aged man. In his chart you notice that he has a pituitary tumor and that he is experiencing some visual deficits due to this. Because a pituitary tumor often affects the optic chiasm, you know he may have difficulty seeing what?

a. Peripherally, so he may best see you if you stand directly in front of him
b. Centrally, so he may not see you if you are standing directly in from of him
c. The top half of you, so he will only see the bottom half of you and his surroundings
d. The bottom half of you, so he will only be able to see the top half of you and his surroundings
e. Anything, he will be completely blind.
a. Peripherally, so he may best see you if you stand directly in front of him. (correct- pituitary tumors are associated with tunnel vision)
b. Centrally, so he may not see you if you are standing directly in from of him.(incorrect- he will see you best when you are directly in front of him)
c. The top half of you, so he will only see the bottom half of you and his surroundings (incorrect- this deficit is related to a lesion of the Meyer’s loop)
d. The bottom half of you, so he will only be able to see the top half of you and his surroundings (incorrect- this is associated with a lesion along the inferior optic radiations)
e. Anything, he will be completely blind (incorrect- he still be able to see directly in front of him, tunnel vision)
3. Louie, a 22 y/o Caucasian male, presents to outpatient physical therapy for an evaluation of a hand injury due to a recent bar fight. Louie’s biggest complaint is that he can no longer see in his lateral visual fields. Your examination confirms that Louie has a visual impairment. What is likely severed/compressed in the visual pathway?

a. Meyer’ Loop
b. L optic Nerve
c. Optic Chiasm
d. R optic tract
e. R optic nerve
a. Meyer’ Loop (incorrect- pt lack superior vision in the contralateral visual field)
b. L optic Nerve (incorrect- pt would lack vision out of the left eye)
c. Optic Chiasm (CORRECT- pt would lack peripheral vision if this was severed)
d. R optic tract (incorrect- pt would lack vision of R medial visual field and L lateral visual field)
e. R optic nerve (incorrect- pt would lack vision of right eye)

Now you have two questions testing the optic chiasm. There are many more visual field deficits to choose from.
4. You are working in an inpatient PT facility and Martha, a 37 y.o Asian female, recently was in a motor vehicle accident and sustained a TBI to the occipital region of the skull (specifically R area 17). You suspect that Martha could have visual deficits. Where in her visual field do you suspect she will be unable to see?

a. Entire R visual field
b. L lateral visual field/ R Medial visual field
c. No visual impairment
d. L medial visual field, R Medial visual field
e. peripheral vision
a. Entire R visual field- (incorrect pt would have this visual impairment if she injured the optic nerve)
b. L lateral visual field/ R Medial visual field (CORRECT- contralateral homonymous heminasopia is associated with injury to R area 17)
c. No visual impairment (Incorrect- area 17 is part of the visual pathway)
d. L medial visual field, R Medial visual field (pt can only see the peripheral) – incorrect L Medial visual field is still intact
e. peripheral vision (incorrect- this impairment is associated with severing of the optic chiasm.

Good question
5. You are working with a 5 year old child in a pediatric clinic. You are having the child do reaching activities towards two stuffed clowns, one on her right and one on her left. You tell her to reach forward and squeeze the nose of the clown on her left. She tells you that only the clown on the right has a face and the clown on the left only has a body. After further investigating you realize she has a serious visual impairment. What is the name of this impairment and what area of the brain might be causing this problem?

a. anopsia; optic tract
b. contralateral homonymous superior quadrantopsia; Meyer’s loop
c. contralateral homonymous hemianosia; optic nerve
d. heteronymous hemianosia; Meyer’s loop
e. None of the above
a. anopsia; optic tract (incorrect - this is blindness, she is only experiencing blindness in one quadrant of her visual field)
b. contralateral homonymous superior quadrantopsia; Meyer’s loop (correct - problems at the Meyer’s loop can cause visual deficits in one quadrant or “pie in the sky” vision)
c. contralateral homonymous hemianosia; optic nerve (incorrect - this would be problems seeing one side of her visual field)
d. heteronymous hemianosia; Meyer’s loop (incorrect - this would be tunnel vision)
e. None of the above

Very good question
6. You are the treating physical therapist for Mr. Thorne-Fitzgenius, a 57 year-old male, who is in the hospital with a chief complaint of headaches from constant bright light. The resident on staff tells you that the headaches are being caused secondary to a strength deficit in the muscle that constricts the pupil. He also knows that strengthening muscles is what PT is all about. What muscle is this misguided resident asking you to strengthen?

a. Lateral Rectus muscle
b. Levator Palpebrae muscle
c. Cilliary muscle
d. Orbicularis Oculi
e. Superior Colliculus
a. Lateral Rectus muscle (Incorrect - this muscle abducts the eye)
b. Levator Palpebrae muscle (Incorrect - this muscle lifts the eyelid)
c. Cilliary muscle (Correct)
d. Orbicularis Oculi (Incorrect - this muscle closes the eyes)
e. Superior Colliculus (Incorrect - not a muscle)

Cute question. He should have referred to vision therapy!
7. You are currently working in an outpatient facility, treating a patient with MS. You’ve been treating this patient for the past two weeks with no incidents, but today she comes in, noting to you that she has had some trouble walking down her stairs in the past 24 hours or so. You ask her about it and she also mentions that she thinks she has double vision. What problem most likely presents itself this way?

a. Cranial nerve 4 palsy
b. Lesion in the Meyer’s loop
c. Lesion at the chiasm
d. Cranial nerve 3 palsy
e. All of the above would lead to double vision
a. Cranial nerve 4 palsy (correct - a palsy of this cranial nerve normally results in vertical diplopia)
b. Lesion in the Meyer’s loop (incorrect - this results in a “pie in the sky” vision in which you lose the top quarter of the visual field.)
c. Lesion at the chiasm (incorrect - this leads to heteronymous hemianopsia, aka loss of peripheral vision)
d. Cranial nerve 3 palsy (incorrect - this would lead to a “down and out” position of the eye.) THIS CAN PRESENT WITH HORIZONTAL DOUBLE VISION
e. All of the above would lead to double vision (incorrect - they would not all lead to vertical diploplia.

You have to make sure that there is only one possible answer. CN3 palsy could present in a similar fashion.
8. Your patient Mac Donald, has been diagnosed as a type II diabetic for 30 years now. He has also had hypertension due to his diabetes. He comes to you asking to help free him of his knee pain (he has a type II obesity rating according to his BMI), but when you ask him some general questions about his well-being, he mentions he’s been having eye problems. You decide to assess the cranial nerves related to the eye and notice he has trouble with lateral eye movements bilaterally. Which cranial nerve is this and what muscle is affected?

a. CN V & lateral pterygoid
b. CN III & superior rectus
c. CN VI & lateral rectus
d. CN XIII & superomedial rectus
e. CN I & orbicularis oculi
a. CN V & lateral pterygoid (Incorrect - this cranial nerve is involved in sensation to the opthalmic region, maxillary region, and motor innervation to the muscles of mastication)
b. CN III & superior rectus (incorrect - this CN is involved in motions other than the one affected)
c. CN VI & lateral rectus (correct - this CN involves abduction of the eyeball)
d. CN XIII & superomedial rectus (incorrect - there is no CN XIII and no such muscle)
e. CN I & orbicularis oculi (incorrect - CN I is the olfactory nerve and is sensory only)

I totally forgot to teach you guys CNXIII… thanks for reminding me!
9. Henry Parker is a 30 year old male that comes to your outpatient physical therapy clinic with a prescription to evaluate and treat neck pain, status post a motor vehicle accident. Because the patient sustained a mild TBI during the motor vehicle accident, you decide to test his cranial nerves. Upon a basic eye evaluation, you can’t help but think of all that is happening in order for Henry to see the visual chart in front of him. Which is the correct sequence of the visual pathway in a healthy individual?

A. Retina, Optic Nerve, Optic Tract, Optic Chiasm, LGN, Visual Cortex, Geniculocalcarine Tract
B.Retina, Visual Cortex, Optic Nerve, Optic Chiasm, Optic Tract, Geniculocalcarine Tract, LGN
C. Retina, Optic Nerve, Optic Chiasm, Optic Tract, LGN, Geniculocalcarine Tract, Visual Cortex
D. Retina, Optic Chiasm, Optic Nerve, LGN, Optic Tract, Geniculocalcarine Tract, Visual Cortex
E. Optic Nerve, Retina, Optic Chiasm, LGN, Geniculocalcarine, Optic Tract, Visual Cortex
A. Retina, Optic Nerve, Optic Tract, Optic Chiasm, LGN, Visual Cortex, Geniculocalcarine Tract (Incorrect) the visual stimulus travels through the Optic Chiasm before the Optic tract, and through the Geniculocalcarine Tract before the Visual Cortex.
B.Retina, Visual Cortex, Optic Nerve, Optic Chiasm, Optic Tract, Geniculocalcarine Tract, LGN (Incorrect) visual stimuli travels through the LGN before the Geniculocalcarine tract and the Visual Cortex is the last area the stimulus travels to.
C. Retina, Optic Nerve, Optic Chiasm, Optic Tract, LGN, Geniculocalcarine Tract, Visual Cortex (Correct) This is the correct sequence of visual input from the retina to the visual cortex.
D. Retina, Optic Chiasm, Optic Nerve, LGN, Optic Tract, Geniculocalcarine Tract, Visual Cortex (Incorrect) The visual stimulus travels through the Optic Tract before the LGN.
E. Optic Nerve, Retina, Optic Chiasm, LGN, Geniculocalcarine, Optic Tract, Visual Cortex (Incorrect) The visual stimulus travels through the Retina first, and through the Optic Tract after the Optic Chiasm.

Good question. I’ll change the stem a little bit.
10. You are working in your outpatient clinic with a 46 year old and performing a CN screen because he has been experiencing dizziness and nausea. While performing the pupillary light reflex test on his left eye, you note that his pupil does not constrict. After clearing CN III through accommodation testing, you suspect a problem with the reflex pathway. What is the correct reflex pathway?

a. Retina, Optic Nerve, Optic Chiasm, Optic Tract, pretectal area, Edinger Westphal nucleus, oculomotor nucleus, oculomotor CN III, ciliary ganglion, postganglionic fibers, ciliary muscle
b. Retina, Optic Nerve, Optic Chiasm, Optic Tract, pretectal area, Edinger Westphal nucleus, oculomotor nucleus, LGN, ciliary ganglion, Area 17, ciliary muscle
c. Retina, Optic Nerve, Optic Tract, pretectal area, oculomotor nucleus, oculomotor CN III, ciliary ganglion, postganglionic fibers, LGN
d. Retina, Edinger Westphal nucleus,Optic Nerve, oculomotor CN III, Optic Chiasm, Optic Tract, pretectal area, oculomotor nucleus, ciliary ganglion, postganglionic fibers, ciliary muscle
e. Retina, Optic Nerve, Optic Chiasm, Optic Tract, pretectal area, Edinger Westphal nucleus, oculomotor nucleus, oculomotor CN IX, ciliary ganglion, postganglionic fibers, ciliary muscle
a. Retina, Optic Nerve, Optic Chiasm, Optic Tract, pretectal area, Edinger Westphal nucleus, oculomotor nucleus, oculomotor CN III, ciliary ganglion, postganglionic fibers, ciliary muscle (correct - this is the correct pathway)
b. Retina, Optic Nerve, Optic Chiasm, Optic Tract, pretectal area, Edinger Westphal nucleus, oculomotor nucleus, LGN, ciliary ganglion, Area 17, ciliary muscle (incorrect - LGN is not part of the reflex pathway, nor is Area 17)
c. Retina, Optic Nerve, Optic Tract, pretectal area, oculomotor nucleus, oculomotor CN III, ciliary ganglion, postganglionic fibers, LGN (incorrect - missing the optic chiasm and LGN is not part of this pathway)
d. Retina, Edinger Westphal nucleus,Optic Nerve, oculomotor CN III, Optic Chiasm, Optic Tract, pretectal area, oculomotor nucleus, ciliary ganglion, postganglionic fibers, ciliary muscle (incorrect - EW cell is much later as is CN III)
e. Retina, Optic Nerve, Optic Chiasm, Optic Tract, pretectal area, Edinger Westphal nucleus, oculomotor nucleus, oculomotor CN IX, ciliary ganglion, postganglionic fibers, ciliary muscle (incorrect - CN IX is not part of the reflex pathway)

Good testing of the concept. I’m going to change the wording around.
Mrs. Jones is a 55 y/o female who is admitted to your clinic s/p ischemic stroke for upper extremity weakness. Both of her arms are weak, as well as her facial muscles. You test her lower extremity muscles and determine that they are not weak. You also notice that her vision is not compromised. Which of the following vessels do you believe was affected by the stroke?

a. ACA
b. MCA
c. PCA
d. PICA
e. All of the above
Answer 1

a. ACA – affects LE not UE
b. MCA – CORRECT; a stroke of the MCA would cause weakness of the UE and the face
c. PCA – the PCA goes to the occipital lobe and would affect her vision
d. PICA – would likely affect the LE as well as the UE
e. All of the above

Why are both her arms weak? I’ll reword the question, most likely.
Mr. Gray is a 60 y/o Caucasian male who comes into your clinic s/p transient ischemic attack (TIA). You notice that he is having trouble with his vision, and he tells you that he has noticed a decline in his vision after he experienced the TIA. Which is the following (GRAMMAR) vessels do you believe could have been affected by the TIA?

a. ICA
b. Basilar artery
c. PCA
d. A and B
e. All of the above
Answer 2

a. ICA – people will first show signs of vision loss because the ophthalmic nerve is the first one that comes off of the ICA
b. Basilar artery – the basilar artery supplies branches that provide bloodflow to the occipital lobe, so a TIA of the basilar artery could cause vision loss
c. PCA – supplies blood to the occipital lobe, which is responsible for vision
d. A and B – C is also correct!
e. All of the above – CORRECT; a TIA of the ICA, PCA or basilar artery could all cause vision loss

Good question. I will modify the wording to not use all of the above or A&B responses.
Mrs. Smith is a 72 y/o Caucasian female who you will be treating s/p SCA ischemic stroke at Udel Hospital. After an extensive cranial nerve exam, you observe that she has difficulty moving her eye in most directions, with the exception of abduction and depression. You conclude that there is most likely damage to CN ____ as a result of its need for dual blood supply by both the SCA & PCA.

a. II
b. III
c. IV
d. V
e. VI
Answer 3

a. II – pt with optic n. damage would likely have vision problems
b. III – CORRECT; the oculomotor nerve is supplied partially by the SCA, as well as the MCA
c. IV – pt with trochlear n. damage would exhibit difficulty with depression of extraocular mm.
d. V – pt with facial n. damage would exhibit difficulty with facial expressions
e. VI – pt with abducens n. damage would exhibit difficulty with abduction of extraocular mm.

CN III pathology is associated with those vessels, however it doesn’t really have to do with the need for a dual blood supply. It has to do with aneurysmal effects on either of those arteries. I’ll reword the question.
Mr. Andrews is a 57 y/o Caucasian male 2 days s/p hemorrhagic stroke who you are seeing in the acute setting. Before you have a chance to look over his chart, you overhear his MD saying that his stroke occurred in a vessel particularly susceptible to rupture due to it’s numerous irregularities. You suspect, therefore, that his stroke could have occurred in all of the following arteries except ________.

a. ICA
b. ACA
c. Pcomm
d. PSA
e. PCA
Answer 4

a. ICA – part of the Circle of Willis and therefore contains branches/irregularities that cause turbulent blood flow leading to compromise; forms junctions with ACA, MCA, and opthalmic aa.
b. ACA – also part of the Circle of Willis; forms junctions with Acom, MCA, & ICA
c. Pcom – part of the Circle of Willis; forms junctions with PCA, MCA, ICA, & ACA
d. PSA – CORRECT; relatively straight pathway with smooth, laminar bloodflow; less likely to be compromised
e. PCA – part of the Circle of Willis; branches into Pcom & SCA

This is difficult to explain, but this is not really a great question… and it was probably my fault. Vessels that are tortuous are more susceptible to atherosclerotic plaque and therefore ischemic strokes, while those that have many branch points are more susceptible to hemorrhage from aneurysmal dilatation. While your question is technically true, in a way, you are comparing apples and oranges. I’ll figure out a way to reword it a little.
Mr. James is a 63 y/o black male s/p ASA ischemic stroke who you are seeing in your first acute care affiliation. You perform a chart review before treating him and note that he is currently experiencing ipsilateral spastic paralysis below the level of the lesion. What portion of the spinal cord does the ASA supply?

a. Anterior 1/3
b. Anterior 2/3
c. Lateral 1/3
d. Posterior 2/3
e. B & C
Answer 5

a. Anterior 1/3 – this is the right direction (ant) but not the correct portion (should be 2/3, not 1/3)
b. Anterior 2/3 – CORRECT; ASA supplies this portion of the spinal cord, with primarily efferent (motor) pathways
c. Lateral 1/3 – the blood supply of the spinal cord is only divided ant/post, not lat/med
d. Posterior 1/3 – this is the supply of the PSA; this section of the spinal cord has primarily afferent (sensory) pathways
e. B & C – b is correct, but c is incorrect due to reasons listed above

I’m going to reword this question a little bit. Good question.
Mr. Reyes is a 62 y/o Hispanic male s/p ischemic CVA who you are seeing in your outpatient clinic for gait and balance re-training. He tells you that among other symptoms, he now is unable to “make different faces” and has experienced some hearing difficulties since the event. You determine that both CN VII & VIII must have been damaged as a result of his CVA. Which vessel(s) were most likely directly occluded?

a. Pontine perforators
b. SCA
c. AICA
d. Labyrinthine aa.
e. C & D
Answer 6

a. Pontine perforators – supply the pons, which CN VII & VIII stem from, but likely do not have as big/direct impact as C & D
b. SCA – supplies the superior cerebellum
c. AICA – CN VII & VIII emerge just above this vessel
d. Labyrinthine aa. – CN VII & VIII emerge just below this vessel
e. C & D – CORRECT; CN VII & VIII emerge btwn these 2 vessels and are therefore dependent on them for blood supply

It’s a good question, but I can see lots of argument on it.
Mr. Somers is a 65 year old Caucasian male referred to your outpatient clinic with numbness and weakness below the level of C7. Imaging for Mr. Somers reveals lateral foraminal disc protrusion at the level of T2. You are concerned about ischemic spinal cord infarction secondary to occlusion of the radicular artery. Which of the following is not a characteristic of radicular arteries?
a. Radicular arteries arise from aorta
b. There are 32 pairs of radicular arteries
c. Radicular arteries bifurcate
d. Supply only the ventral portion of the spinal cord
e. All of the above
Answer 7

a. Radicular arteries arise from aorta - radicular arteries arise from the intercostal arteries off of the aorta
b. There are 32 pairs of radicular arteries - there is a total of 32 pairs of radicular arteries
c. Radicular arteries bifurcate - radicular arteries bifurcate so that they can supply both dorsal and ventral aspects of the spinal cord
d. Supply only the white matter of the spinal cord - CORRECT; this is NOT a characteristic of radicular arteries. Radicular arteries supply both white and grey matter of the ventral and dorsal portions of the spinal cord.
e. All of the Above - a,b, and c are all features of radicular arteries but d is not

Good question… on the more difficult side.
Mr. Cohen is a 66 y/o male s/p ischemic CVA referred to your outpatient clinic for gait retraining. Due to his balance and coordination deficits, you suspect involvement of the Cerebellum. Which of the following arteries were most likely affected by the CVA?
a. Right MCA
b. Left SCA
c. Left ICA
d. ASA
e. None of the above
Answer 8

a. Right MCA - the right Middle Cerebral Artery supplies blood to the right lateral cerebral cortex, as well as the anterior temporal lobes and insular cortices.
b. Left SCA - CORRECT; the left Superior Cerebellar Artery supplies blood to the upper left surface of the Cerebellum
c. Left PCA - the left Posterior Cerebral Artery supplies blood to the left posterior aspect of the occipital lobe
d. ASA - the Anterior Spinal Artery supplies blood to the ventral ⅔ of the spinal cord
e. None of the above - a, c, and d do not supply the Cerebellum, but b does and is the correct answer

Great question. I’ll reword slightly.
Dr. Roberts is a 72 y/o female who you are treating in your inpatient rehab clinic for sensory and motor deficits secondary to Cranial Nerve Disease resulting from an injury at the level of her pons. What are possible motor or sensory deficits you would expect to see if only the Cranial Nerves originating form the pons were affected?
a. Weakness of bilateral SCM
b. Weakness of muscles involved in facial expression
c. Weakness in the muscles of mastication
d. “Down and Out” gaze
e. B & C
Answer 9

a. Weakness of bilaterally SCM - SCM is innervated by CN XI, originating from the medulla
b. Weakness of muscles involved in facial expression - Muscles of facial expression are innervated by CN VII, which does originate in the pons
c. Weakness in the muscles of mastication - muscles of mastication are innervated by CN V, which does originate in the pons
d. “Down and Out” gaze - “Down and Out” gaze is associated with weakness of the Superior Oblique muscle, which is innervated by CN IV, originating from the midbrain
e. B & C - CORRECT; Both B & C are characteristics associated with CN’s that originate from the pons

Great question. I’ll reword it a little bit.
Mrs. Thomas is a 57 y/o female who you are treating in your outpatient clinic s/p ischemic stroke. She presents with loss of ipsilateral vibration and joint position sense. Which of the following vessels do you believe was affected by her stroke?

a. PSA
b. AICA
c. PICA
d. Acomm
e. SCA
Answer 10

a. PSA - CORRECT; because the PSA is located in the dorsal part of the spinal column, it will be responsible for sensory changes
b. AICA - supplies blood to the anterior, inferior cerebellum, which would cause deficits in movement coordination and balance if this vessel was affected in a stroke
c. PICA - supplies blood to the posterior, inferior cerebellum, which would cause deficits in movement coordination and balance if this vessel was affected in a stroke
d. Acomm - part of the circle of Willis in the midbrain that connects the left and right ASA; would not affect vibration and joint position if this was the only vessel affected by the stroke
e. SCA - supplies blood to the superior cerebellum, which would cause deficits in movement coordination and balance if this vessel was affected by the stroke

Good question. Actually, sometimes a diagnostic dilemma because all of these strokes will present with balance deficits.
An 18 y.o. male basketball sustained a traumatic blow to his head while attempting to grab a rebound during a game two weeks ago. When he arrived to the hospital, he was diagnosed with a mild concussion and since then his sx have subsided. He was referred to PT for a graded exercise return to sport. During the initial PT evaluation, the patient c/o difficulty with swallowing and slurred speech. A cranial nerve exam was performed, and it was found that the patient’s tongue deviated to the right when he was asked to stick it out. This abnormal finding is indicative of an injury to which cranial nerve?

a. CN IX
b. CN VII
c. CN V
d. CN XII
e. CN II
Answer 11

a. CN IX (incorrect b/c cranial nerve IX only sensory taste of the posterior 1/3 of tongue)
b. CN VII (incorrect b/c CN VII is responsible for innervating the muscles of facial expression)
c. CN V (incorrect b/c CN V is only responsible for innervating the muscles of mastication)
d. CN XII (CORRECT; damage to CN XII results in atrophy of the genioglossus on the ipsilateral side of the injury, resulting in deviation of the tongue to the injured side with protrusion)
e. CN II (Incorrect b/c CN III only controls eye movements)

I’m going to change the wording of this question to also test the side of the lesion. Also, a very important concept!
A 36 y/o Caucasian female arrived at outpatient PT with complaints of difficulty swallowing and loss of taste on the posterior ⅓ of the tongue. Which CN is most likely impaired?

a. CN V
b. CN VII
c. CN IX
d. CN X
e. CN XII
Answer 12

a. CN V (incorrect b/c CN V innervates sensory not taste in tongue)
b. CN VII (incorrect b/c CN VII innervates anterior ⅔ of tongue)
c. CN IX (CORRECT; CN IX innervates the posterior 1/3 of tongue and motor for swallowing)
d. CN X (incorrect, does innervate motor for swallowing but not taste in tongue)
e. CN XII (incorrect, innervates motor component of tongue)

Good question
A 76 y/o Caucasian male reported to the outpatient PT clinic with complaints of cervical pain. Through CN screening it was evident that the pt was unable to visually track laterally with his R eye. Which CN is likely the cause?

a. CN VI
b. CN IV
c. CN III
d. CN VIII
e. CN II
Answer 13

a. CN VI (CORRECT; CN VI Abducens innvervates the lateral rectus which abducts the eye)
b. CN IV (incorrect b/c Trochlear innervates the superior oblique which depresses and rotates medially)
c. CN III (incorrect, all eye movements but lateral and inferomedial)
d. CN VIII (incorrect, we haven’t even covered this one yet)
e. CN II (incorrect, optic not occulomotor)

Good question
A 34 y/o male is being seen in an outpatient PT clinic for a sprained ankle after a fall playing basketball. Pt does not remember what happened and reports he may have lost consciousness. While obtaining the pt’s medical history you notice that his head is tilted. He also reports that he has been noticing some issues with going up and down stairs secondary to difficulty seeing the stairs. Suspecting that the patient may have suffered a concussion you examine him further and find that he has difficulty looking downward. Which CN is most likely the cause of this pt’s symptoms?

a. CN II
b. CN III
c. CN IV
d. CN V
e. CN VI
Answer 14

A. CN II (incorrect; optic nerve is not involved in visual tracking) – ACTUALLY IT’S THE AFFERENT INPUT!
B. CN III (incorrect; oculomotor nerve does not innervate the superior oblique muscle involved in a downward gaze)
C. CN IV (CORRECT; weakness of downward gaze (superior oblique), head tilt helps to line up the double vision that is experienced, vertical diplopia causes difficulty ascending/descending stairs)
D. CN V (incorrect; trigeminal nerve is not involved with vision/eye movements)
E. CN VI (incorrect; abducens nerve is responsible for the motor of the lateral rectus muscle which would abduct they eye)

Good question
A 44 y/o AA female arrived at the outpatient PT clinic with complaints of dizziness and severe headaches. During CN screening you discover, using the pupillary light reflex, that she is unable to constrict her cilliary muscles. In which part of the brain is the CN that is likely responsible for her deficiency located?

a. Cerebellum
b. Prosencephalon
c. Midbrain
d. Pons
e. Medulla
Answer 15

a. Cerebellum (Incorrect, no CN’s located in the cerebellum)
b. Prosencephalon (Incorrect, CN’s I and II)
c. Midbrain (CORRECT; CN III is located in the midbrain)
d. Pons (Incorrect, CN’s V-VIII)
e. Medulla (Incorrect, CN’s IX-XII)

Good question… I’ll reword it slightly.
After a very long and hard neuroanatomy test you and your friends decide to head to Grotto’s to grab a beer and relax. Your friend has a little bit too much to drink and falls off the bar stool while trying to catch the popcorn you are throwing in his mouth. On his way to the floor he hits his head on the table and falls onto his left shoulder but gets right back up and keeps drinking. The next day he has a headache (which he attributes to a hangover) but mainly he is noticing that he is having trouble elevating his right arm. You ask him to shrug his shoulders and notice that only his left shoulder is elevating. You decide to also check his SCM and find that his right SCM is a 0/5. Which CN do you suspect was injured?

a. CN III
b. CN IX
c. CN X
d. CN XI
e. CN XII
Answer 16

a. CN III (incorrect; occulomotor is responsible for eye movements)
b. CN IX (incorrect; glossopharyngeal provides sensation to the palate, posterior 1/3 of the tongue, pharynx and middle ear cavity
c. CN X (incorrect; Vagus innervates muscles of the soft palate, pharynx and larynx)
d. CN XI (CORRECT; spinal accessory innervates the upper traps and SCM)
e. CN XII (incorrect; hypoglossal is responsible for tongue protrusion)

I’m guessing there’s a story behind this question…
You are at a high school football game watching your younger brother play. After a nasty hit you notice a kid having trouble getting up and appears dazed. There are no other medical personnel present and you believe something could be wrong so you decide to go check the kid out. He reports he has a headache, is dizzy and feels nauseous so you suspect a concussion. You perform a cranial nerve exam on him. His eyes do not accommodate to shining a pen light (WHAT DOES THIS MEAN???) in his eye and he has trouble tracking superiorly, inferiorly and medially. Which CN(s) do you suspect could be injured?

a. CN III
b. CN IV
c. CN VI
d. A and B only
e. All of the above
Answer 17

a. CN III (incorrect; both a and b)
b. CN IV (incorrect; both a and b)
c. CN VI (incorrect; abducens, not having trouble abducting the eye)
d. A and B only (CORRECT; pt is having trouble looking superior and medially and is not accommodating to light (CN III) and is having trouble looking inferiorly (CN IV))
e. All of the above. (incorrect; not having trouble with abducting the eye)

Accommodation doesn’t have to do with a pen light. You also should specify which eye…
I’ll reword the question.
In the scenario above, the accommodation reflex is tested during the cranial nerve exam. Which of the following is not a result of the accommodation reflex?
a. His lens’ will round up
b. The boy’s pupils will constrict
c. The boy’s eyes will converge
d. The boy’s eyes will diverge
e. When he closes his eyes, his eyes will diverge
Answer 18

a. His lens’ will round up (incorrect; The lens’ round up in conjunction with pupillary constriction and his eyes will converge to fixate on an object moving towards him)
b. The boy’s pupils will constrict (incorrect; see the explanation above)
c. The boy’s eyes will converge (incorrect; See the explanation for answer A)
d. The boy’s eyes will diverge (CORRECT; the eyes will diverge when the boy closes his eyes b/c the eyes will not be focusing on any particular object)
e. When he closes his eyes , his eyes will diverge (incorrect; When the eyes are closed the pupils will dilate and the eyes will diverge because there are no objects to fixate on)

This question was not submitted in the correct format. What happened? Good question overall.
A 52 y.o. wm reports to the hospital with an infarct of the Superior Cerebellar Artery (SCA).(INFARCTS DON’T AFFECT VESSELS, THEY AFFECT PARTS OF THE BRAIN… YOU CAN SAY AN SCA DISTRIBUTION) During the physical examination the next day, the PT realizes that the man has trouble lifting his right eyelid, a symptom common of the condition called ptosis. What Cranial nerve was most likely damaged and the cause of the ptosis?

a. CN IV
b. CN III
c. CN V
d. CN VI
e. CN VII
Answer 19

a. Cranial Nerve IV (incorrect; The trochlear nerve innervates the superior oblique muscle which depresses the eyeball)
b. Cranial Nerve III (CORRECT; CN III innervates the levator papelbrae muscle that elevates the eyelid
c. Cranial Nerve V (incorrect; innervates the muscles of mastication)
d. Cranial Nerve VI (incorrect; innervates the lateral rectus which abducts the eyeball)
e. Cranial Nerve VII (incorrect; innervates the muscles of facial expression, in control of active eye closing)

Good question
It’s a typical Saturday night at Kate’s and one of our classmates is dancing. As always it is crowded and he is pushed down the stairs, hitting his head along the way. The next morning he experiences some dysphonia but thinks it is from drinking and staying up late. However, two days later he is still experiencing a headache and is sensitive to light. Just learning cranial nerves you do an assessment. When you asked him to say “ah” you notice that his uvula deviates to the left. This person most likely has a lesion of the axons of CN ______ on the _______ side?

a. X, Left
b. X, Right
c. XII, Left
d. XII, Right
e. None of the above
Answer 20

a. X, Left (incorrect; Vagus nerve is correct but uvula does not deviate ipsilaterally)
b. X, Right (CORRECT; Vagus nerve innervates the soft palate--uvula; uvula deviates contralateral to the side of the lesion)
c. XII, Left (incorrect; Hypoglossal is responsible for motor of the tongue)
d. XII, RIght (incorrect; Hypoglossal is responsible for motor of the tongue)
e. None of the above (incorrect; X on the right is the correct answer)

Excellent question. I’ll reword this slightly.
You are on your acute care affiliation at the local hospital and your CI wants you to go down to the NICU (neonatal ICU) to observe a specific baby suffering from a heart defect whose mother is an alcoholic, partaking (is she still pregnant???-grammar!) in binge drinking while pregnant. It is likely that this child will experience other problems including neurodevelopmental delay. Which is true?

a) Alcohol does not play a role in this infant’s condition.
b) The fetus’s blood:brain barrier was not fully formed, thus allowing whole blood, carrying alcohol, to reach the neurons and glia damaging them.
c) It is unlikely that the child will have any facial deformities.
d) Your CI is weird and thinks you are an alcoholic and does not want your future babies to turn out like this.
e) Despite the blood:brain barrier not being fully formed, the other barriers will hinder the alcohol’s effects.
Answer 1
1. Alcohol consumption during pregnancy (especially excessive amounts) can cause fetal alcohol syndrome (FAS), which can lead to neurodevelopmental delay.
2. Correct.
3. Signs of FAS include facial characteristics such as a very thin upper lip, short, upturned nose, etc.
4. While your CI may think you are an alcoholic, this has nothing to do with the baby’s condition.
5. The three fluid systems in the brain (vascular, CSF, extracellular) are compartmentalized and do not communicate freely.

This might be too much of a give away question. If you read the choices, without reading the question, you can still deduce the answer. I’m glad you asked about FAS and I will include it on the test, but I will reword the question.
Question 3
You are performing a cranial nerve exam on your patient Mrs. Magee, age 58. You ask her to show you her teeth and when she does so, you can only clearly see her top and bottom teeth on her L side of her mouth – she seems to have trouble showing you her teeth on her R side. However, when you ask her to raise her eyebrows, she does so without a problem. You also notice that each time you perform a R patellar reflex Mrs. Magee kicks you (reflex score: 3+). You suspect:

a) your patient may have experienced a left cortical stroke
b) she may be experiencing a left supranuclear facial palsy
c) Mrs. Magee does not have a cranial nerve problem. She kicks you because you called her Big Boobs Magee earlier and she is not unable to show you her teeth, she is just sneering at you.
d) that Mrs. Magee is showing the signs of Bell’s Palsy
e) both a and b
Answer 3
1. See #5 below.
2. See #5 below.
3. This is an example of being unprofessional towards your patient.
4. Bell’s Palsy results from a lower motor neuron (LMN) lesion and her reflex indicate upper motor neuron (UMN) lesion.
5. Correct. Both a and b are possibilities that indicate an UMN lesion as well as paralysis of the lower ¼ of the face seen here.

This is a good question. I will reword it a little. I will probably also eliminate choice C in order to keep my job.
Mrs. Finnigan, age 30, comes into your clinic. She is currently 7 months pregnant. She complains of “not being able to eat right.” You notice that her facial expressions seem to be diminished, so you decide to do some cranial nerve testing. When testing her facial nerve (CN VII) you ask her to raise her eyebrows and only her L eyebrow raises. When you ask her to smile, you can only see her teeth on her L side. You suspect:

a) your patient may have experienced a left cortical stroke
b) she may be experiencing a left supranuclear facial palsy
c) Mrs. Finnigan, in one of her weird cravings, ate something funny and is now having an allergic reaction.
d) that Mrs. Finnigan is showing signs of Bell’s Palsy
e) both a and b
Answer 4
1. Both this answer and answer #2 or b refer to upper motor neuron (UMN) lesions in which only the lower half, of half of the face, is paralyzed.
2. See above.
3. If you assume that this is just an allergic reaction, you will have a very dissatisfied patient.
4. Correct. Paralysis of the muscles of the entire half of the face indicates Bell’s Palsy.
5. See #1 above.

Oddly enough, that could be a sign of an allergic reaction, as can acute lower back pain! I’ll reword the question a little but good overall.
You are currently in the hospital visiting Sam, a 15 year old patient who just experienced a TBI in an automobile accident that knocked him unconscious. Now awake two days later, you notice that Sam is tilting his head and tucking his chin into his chest when he talks to you. You ask him to read a newspaper title, but he says that he sees two titles. You suspect Sam has a specific nerve palsy.


2. What muscle (s) is (are) denervated with the palsy you suspect?
a. Inferior rectus
b. Inferior oblique
c. Superior oblique
d. Medial rectus
e. B and D
Answer 2
a. Wrong. This is CN III
b. Wrong. This is CN III
c. CORRECT. This is CN IV
d. Wrong. This is CN III
e. Wrong. Although it this may present with a similar “down and out” presentation, both are associated with CN III

Good
Mr. Caffery is a 34 year old male. One week ago, he reported that he got paresthesia in his toes. However, you notice that the weakness spread very rapid (GRAMMAR) in the past week that sensory impairments and symmetric motor weakness is shown in both his legs and arms. As his PT, you suspect Guillain-Barre’ Syndrome in this male because of his impaired peripheral nerve integrity, progressive weakness in more than one extremity, and symmetric weakness.

Question 1.
In this case, which of the following type of glial cell is the main target of attack by the antibody mediated demyelination?
a. Oligoendroglia Cells
b. Ependymal Cells
c. Microglial Cells
d. Schwann Cells
e. Purkinje Cells

Good question overall. “got” is never an acceptable written word… did he “report parasthesias” or “report experiencing parasthesias” or “report developing parasthesias”
Answer 1.
a. Form segments of myelin on axons in CNS, not PNS.
b. Epithelial cells that line the CSF-filled ventricles in the brain and the central canal of the spinal cord, not in PNS
c. Glial cell that are the resident macrophages of the brain and spinal cord, not PNS
d. Correct. Found in PNS. Schwann cells form a neurolemma sheath around axons.
e. Found in cerebellum, not PNS
paresthesia in his toes. However, you notice that the weakness spread very rapid (GRAMMAR) in the past week that sensory impairments and symmetric motor weakness is shown in both his legs and arms. As his PT, you suspect Guillain-Barre’ Syndrome in this male because of his impaired peripheral nerve integrity, progressive weakness in more than one extremity, and symmetric weakness.


If the demyelination occurs in CNS instead of PNS, which cell is more likely to be damaged?
a. Oligoendroglia Cells
b. Ependymal Cells
c. Microglial Cells
d. Schwann Cells
e. Purkinje Cells
Answer 2
a. Correct. Form segments of myelin on axons in CNS.
b. Epithelial cells that line the CSF-filled ventricles in the brain and the central canal of the spinal cord
c. Glial cell that are the resident macrophages of the brain and spinal cord
d. Found in PNS. Schwann cells form a neurolemma sheath around axons.
e. Found in cerebellum

Good
Ms. Palmer is a 30 year old female. She is currently treated as a patient for left hand weakness. Recently, she says she started to have diplopia and balance problems. According to these descriptions, you suspect that she might have Multiple Sclerosis (MS).

Question 3.
MS symptoms may be caused by:
a. Damage to myelin
b. Damage to axons
c. Damage to NEURON cell bodies
d. Both a & b
e. Both a & c
Answer 3
a. MS is categorized as a demyelinating disorder which damages the myelin sheath
b. Severe axonal loss is also found is MS, even early in the development of the MS.
c. NEURON Cell bodies were not damaged in MS. The autoimmune system only damages the connections between cell bodies.
d. Correct
e. c is not correct.

I added NEURON because damage may be occurring at the cell body of the oligodendrocyte.
Answer 3
a. MS is categorized as a demyelinating disorder which damages the myelin sheath
b. Severe axonal loss is also found is MS, even early in the development of the MS.
c. NEURON Cell bodies were not damaged in MS. The autoimmune system only damages the connections between cell bodies.
d. Correct
e. c is not correct.

I added NEURON because damage may be occurring at the cell body of the oligodendrocyte.


Question 4
The lesion of MS can be found in:
a.cerebral cortex
b.spinal cord
c.cerebellum
d.basal ganglia
e. all of above
Answer 4.
The sclerotic plaques disseminated throughout the CNS. All are correct.

Good
Mrs. Smith is a 28 year old Caucasian female who recently gave birth to a cute baby girl. You are currently treating baby Smith in pediatric clinic when she is diagnosed with a high fever without an unknown source.
As her PT, you suspect that baby Smith could have meningitis, so you need to sample her CSF.
Which of the following is a relatively safe point for needle isertion to sample CSF?

a. Between C7 and T1
b. Between T12 and L1
c. Between L1 and L2
d. Between L3 and L4
e. At the cervical region
Answer 1

a. Between C7 and T1 – above conus medullaris is not safe for lumbar puncture as the spinal cord may be damaged
b. Between T12 and L1 - above conus medullaris is not safe for lumbar puncture as the spinal cord may be damaged
c. Between L1 and L2 - above conus medullaris is not safe for lumbar puncture as the spinal cord may be damaged
d. Between L3 and L4 – correct: anywhere below the conus medullaris is safe for lumbar puncture
e. At the cervical region - above conus medullaris is not safe for lumbar puncture as the spinal cord may be damaged
Mr. Gunther is a 20-year-old college student who was involved in a car accident and suffered a Spinal Cord Injury at T4. He has a Foley catheter but a nurse inadvertently stepped on it and partially pulled it out. Nobody is aware of this and you, the PT, come into his room and should expect all of the following symptoms except:
a. Pounding headache
b. Nasal congestion
c. Increased heart rate
d. Restlessness
e. Blotching of the skin
Answer 6
a. Pounding headache – symptom of Autonomic Dysreflexia
b. Nasal congestion - symptom of Autonomic Dysreflexia
c. Increased heart rate – correct answer, this is not a symptom of Autonomic Dysreflexia. You should see a slow pulse
d. Restlessness - symptom of Autonomic Dysreflexia
e. Blotching of the skin - symptom of Autonomic Dysreflexia
Waldo Waffle is a 34 year-old weekend warrior at your outpatient clinic receiving treatment for a right ACL tear from playing tackle football with his friends. While at the clinic a ceiling tile falls from the ceiling and narrowly misses hitting him on the head. What physiologic response would you expect to see from Mr. Waffle?
a. Increased watery salivary secretions.
b. Increased viscous salivary secretions.
c. Miosis
d. Bronchoconstriction
e. Vasodilation of renal vascular bed
Answer 7
a. Increased watery salivary secretions. – Parasympathetic response
b. Increased viscous salivary secretions. – Correct, we are looking for a sympathetic response
c. Miosis – Parasympathetic response
d. Bronchoconstriction – Parasympathetic response
e. Vasodilation of renal vascular bed – Parasympathetic response
You are working in an outpatient sports and orthopedics clinic, and your 37 year old female patient who has type II diabetes, complains of numbness and tingling in both hands and both feet. Which axons carry this sensory information to the central nervous system?
1. The fasiculus gracilis tract of the dorsal column
2. The fasiculus cuneatus tract of the dorsal column
3. The spinothalamic tract of the lateral column
4. Both 1 and 2
5. The anterior spinocerebellar tract of the ventral column
Answer 10
1. The fasiculus gracilis tract of the dorsal column – true, but lower extremities, not upper
2. The fasiculus cuneatus tract of the dorsal column – true but for upper extremities, not lower
3. The spinothalamic tract of the lateral column – false – these carry sensory information, but not specifically from the hands or feet.
4. Both 1 and 2
5. The anterior spinocerebellar tract of the ventral column – false – conveys proprioceptive information, not sensory.
Paula is a 33 year old marathoner. According to her, the human brain is insensitive to pain. As a PT well-trained in University of Delaware, which of the following is NOT true regarding ascending pathway in human CNS?
a. All sensory information enter spinal cord at dorsal horn
b. The free nerve endings in anterolateral part are unmyelinated
c. Anterolateral part of ascending pathway only transmit dull and poorly localized pain
d. Lesion at dorsal column will cause loss sensation on the same site of body
e. Trigeminal pathway is involve in sensation of temperature, joint position, pain and
discriminative touch
Answer 1
a. All sensory information enter spinal cord at dorsal horn - true
b. The free nerve endings in anterolateral part are unmyelinated - true, it is slow conduction
nerves
c. Anterolateral part of ascending pathway only transmit dull and poorly localized pain –correct:
they also transmit sharp and well localized pain
d. Lesion at dorsal column will cause loss sensation on the same site of body - true
e. Trigeminal pathway is involve in sensation of temperature, joint position, pain and
discriminative touch - true
Mr. Chuck Wagner is a 44-year-old male PT patient you are treating for low back pain. While Chuck was doing therapeutic exercises a bee flies in through the window and stings him on his thigh. What “path” will the sensation from the bee sting travel through?
a. Dorsal column system
b. Trigeminal pathway
c. Anterolateral system
d. Lateral coritcospinal/pyramidal system
e. Transverse column system
Answer 3
a. Dorsal column system – Discriminative touch, joint position, & vibration
b. Trigeminal pathway – Discriminative touch, joint position & vibration from face
c. Anterolateral system – correct, Pain, temp and crude touch goes through here
d. Lateral coritcospinal/pyramidal system – Descending pathway providing fine motor control to limbs and digits
e. Transverse column system – Does not exist
Mrs. Blairfore is a 32-year-old female office worker who strained her calf while going up a flight of stairs. You are treating her at the U of D Clinic and are using TENS to help with her calf pain. Which system are you targeting with this modality?
a. Dorsal column system
b. Trigeminal pathway
c. Anterolateral system
d. Lateral coritcospinal/pyramidal system
e. Transverse column system
Answer 4
a. Dorsal column system – correct, Discriminative touch, joint position, & vibration
b. Trigeminal pathway – Discriminative touch, joint position & vibration from face
c. Anterolateral system – Pain, temp and crude touch goes through here
d. Lateral coritcospinal/pyramidal system – Descending pathway providing fine motor control to limbs and digits
e. Transverse column system – Does not exist
You are treating a 34 year old male patient for shoulder pain and upper trap tightness. You noticed that the patient is under a lot of stress from deadlines at work and cannot properly perform his HEP due to abdominal pains caused by indigestion. You suspect that stress is playing a major role because:

a. Increased activity of myenteric plexus
b. Increased activity of Meissner’s plexus
c. Sympathetic override of the intrinsic activity
d. Parasympathetic override of the intrinsic activity
e. Increased activity of submucosal plexus
Answer 5
a. Increased activity of myenteric plexus – increase in activity leads to increased digestive tract motility
b. Increased activity of Meissner’s plexus – increase in activity leads to increased absorption of water & ions
c. Sympathetic override of the intrinsic activity – correct - inhibits peristalsis & secretions of glands
d. Parasympathetic override of the intrinsic activity – elicit peristalsis & secretions
e. Increased activity of submucosal plexus – this is another term for Meissner’s plexus, and would therefore increase digestion
Tomorrow you have your first neuro patient. You know that she is hemiplegic due to an MVA that injured a part of her spinal cord. Which of the following tract(s) might have been severed?
a. Ipsilateral spinothalamic tract
b. Contralateral spinothalamic tract
c. Ipsilateral cortiospinal tract
d. Contralateral corticospinal tract
e. A & B
Answer 6
a. Ipsilateral spinothalamic tract – sensory tract
b. Contralateral spinothalamic tract – sensory tract
c. Ipsilateral cortiospinal tract – leads to contralateral hemiplegia
d. Contralateral corticospinal tract – correct - leads to contralateral hemiplegia
e. A & B
At which level does the corticospinal tract cross over?
a. Spinal cord
b. Pons
c. Medulla
d. Midbrain
e. None, it doesn’t cross over
Answer 7
a. Spinal cord – contralateral (with slight ipsilateral) at this level
b. Pons – ipsilateral at this level
c. Medulla – correct – this is the level of the decussation of pyramids.
d. Midbrain – ipsilateral at this level
e. None, it doesn’t cross over
The chart for one of your SCI patients indicates possible damage to the anterolateral system. This finding is further confirmed by a lack of pain & temperature sensation. What other sensation(s) might be missing based on this finding?
a. Vibration
b. Crude touch
c. Discriminative touch
d. Pressure
e. A & D
Answer 8
a. Vibration – dorsal column
b. Crude touch – correct - anterolateral column
c. Discriminative touch – dorsal column
d. Pressure – dorsal column
e. A & D
You are working in an acute care hospital. Your patient, a 25 year old male, was recently in a motor vehicle collision. His vital signs are stable, and he states he is ready to get out of bed because he wants to go to the bathroom on his own. In fact, his exact words were “My gastro-colic reflex is faster than my knee jerk reflex!” How would you disprove this to him?
a. Explain to him that the somatic nervous system is faster acting than the visceral nervous system
b. Explain to him that the visceral nervous system is faster acting than the sensory nervous system
c. Have him eat a burrito and while he is doing this perform a knee-jerk reflex to see which elicits a response first
d. Explain to him that the slowest sensory system is the visceral afferent system, and that the fastest sensory system is the somatic efferent system
e. Both 1 and 4 are correct.
Answer 9
a. Explain to him that the somatic nervous system is faster acting than the visceral nervous system- correct, but combined with 4 this answer makes even more sense
b. Explain to him that the visceral nervous system is faster acting than the sensory nervous system – this is incorrect – this system is slower acting than the sensory nervous system
c. Have him eat a burrito and while he is doing this perform a knee-jerk reflex to see which elicits a response first – this is difficult to measure, and unreliable. Also, there are no taquerias in acute care hospitals.
d. Explain to him that the slowest sensory system is the visceral afferent system, and that the fastest sensory system is the somatic efferent system – this is true, and combined with choice one makes the most sense.
e. Both 1 and 4 are correct.